Sunteți pe pagina 1din 52

Join Our Telegram Channel https://t.

me/UPSCMaterials For Instant Updates

1. The Fundamental Rights are ordinary legal remedies. Hence,


enshrined in Part III of the Indian statement (2) is correct.
Constitution from Articles 12 to 35. The scope of operation of Fundamental
With reference to the Fundamental Rights is limited by Article 31A (saving of
Rights, consider the following laws, providing for acquisition of estates,
statements: etc.), Article 31B (validation of certain
(1) Only the Parliament can acts and regulations included in the 9th
make law for giving effect to Schedule) and Article 31C (saving of
the Fundamental Rights. laws giving effect to certain directive
(2) When the Fundamental principles).
rights, available against the Hence, statement (3) is correct.
State’s action only, are Source: Indian Polity- M Laxmikanth, 4th
violated by the private Edition ch-7; Fundamental Rights; page-
individuals, there are no 7.2
constitutional remedies
available. 2. With respect to the change in name
(3) The scope of operation of of any State in India, consider the
Fundamental Rights is limited following statements:
by Article 31A, Article 31B (1) A Bill contemplating the
and Article 31C. change can be introduced in
Which of the statements given the Parliament only with the
above are correct? prior recommendation of the
(a) 1 and 2 only President.
(b) 1 and 3 only (2) Before recommending the
(c) 2 and 3 only Bill, the President has to refer
(d) 1, 2 and 3 the same to the State
Legislature concerned for
Answer: (d) expressing its views within a
Explanation: specified period.
Most of the fundamental rights are Which of the statements given
directly enforceable (self-executory) above is/are correct?
while a few of them can be enforced on (a) 1 only
the basis of a law made for giving effect (b) 2 only
to them. Such a law can be made only by (c) Both 1 and 2
the Parliament and not by State (d) Neither 1 nor 2
Legislatures so that uniformity throughout
the country is maintained (Article 35). Answer: (c)
Hence, statement (1) is correct. Explanation:
Most of the Fundamental Rights are Article 3 of the Constitution of India
available against the arbitrary action of authorises the Parliament to:
the State, with a few exceptions like (a) form a new State by separation of
those against the State’s action and territory from any State or by uniting two
against the action of private individuals. or more States or parts of States or by
When the rights which are available uniting any territory to a part of any State,
against the State’s action only are (b) increase the area of any State,
violated by the private individuals, there (c) diminish the area of any State,
are no constitutional remedies but only (d) alter the boundaries of any State, and

1
Prelim IAS Test Series (2019) – GS Test 14 (17.03.2019)
GS (Paper-I) Full Test 1
Join Our Telegram Channel https://t.me/UPSCMaterials For Instant Updates

(e) alter the name of any State.


However, Article 3 lays down two Answer: (a)
conditions in this regard: Explanation:
(1) A Bill contemplating the above The Intergovernmental Panel on Climate
changes can be introduced in the Change (IPCC) has recently released a
Parliament only with the prior special report on the impacts of global
recommendation of the President; warming of 1.5 °C above pre-industrial
(2) Before recommending the Bill, the levels and related global greenhouse gas
President has to refer the same to emission pathways.
the State Legislature concerned for IPCC is the international body for
expressing its views within a assessing the science related to climate
specified period. change. The IPCC was set up in 1988 by
The President (or the Parliament) is not the World Meteorological Organization
bound by the views of the State (WMO) and United Nations Environment
Legislature and may either accept or Programme (UNEP) to provide
reject them, even if the views are policymakers with regular assessments
received in time. Further, it is not of the scientific basis of climate change,
necessary to make a fresh reference to its impacts and future risks, and options
the State Legislature every time an for adaptation and mitigation. Hence,
amendment to the Bill is moved and statement (1) is correct.
accepted in the Parliament. The IPCC works by assessing published
Hence both the statements are literature and it does not conduct its own
correct. scientific research. Hence, statement (2)
Source: Indian Polity- M Laxmikanth, 4th is incorrect.
Edition Ch-5; Union and Its Territory; Ref:https://www.ipcc.ch/site/assets/up
page- 5.2 loads/2018/02/FS_what_ipcc.pdf
News:
3. Consider the following statements https://indianexpress.com/article/expl
with reference to the ained/global-warming-how-to-reach-a-
Intergovernmental Panel on 1-5-degree-world-5390972/
Climate Change (IPCC):
(1) The IPCC was set up in 1988 4. The term “White Helmets”, often
by the World Meteorological seen in the news, is related to:
Organization (WMO) and (a) The United Nations'
United Nations Environment campaign for ensuring road
Programme (UNEP). safety across the world.
(2) IPCC conducts its own (b) An international NGO
scientific research to find providing humanitarian
scientific basis of climate assistance to people living in
change, its impacts and war-torn areas.
future risks. (c) A Government of India
Which of the statements given initiative to produce low cost,
above is/are correct? affordable helmets to ensure
(a) 1 only road safety.
(b) 2 only (d) A volunteer corps of Syrians
(c) Both 1 and 2 who act as first responders in
(d) Neither 1 nor 2 the Syrian civil war.

2
Prelim IAS Test Series (2019) – GS Test 14 (17.03.2019)
GS (Paper-I) Full Test 1
Join Our Telegram Channel https://t.me/UPSCMaterials For Instant Updates

Explanation:
Answer: (d) Sl. Aspects Himalayan Peninsul
Explanation: No River ar River
The Syrian Civil Defence, known .
commonly as the White Helmets, is a 1. Place of Himalayan Peninsula
volunteer corps of Syrians who act as Origin mountain r plateau
first responders in the Syrian civil war. covered and
Established in 2013, the group’s charter with central
is to carry out search-and-rescue glaciers highland
operations to save the maximum number 2. Nature of Perennial; Seasonal;
of lives. Inspired by a Quranic verse that Flow receive dependen
says “to save a life is to save all of water from t on
humanity,” the group has rescued more glacier and monsoon
than 60,000 people - a feat that earned rainfall rainfall
them the Right to Livelihood Award, 3. Type of Anteceden Super
commonly known as the “Alternative Drainage t and imposed,
Nobel Prize,” in recognition for their consequen rejuvenat
“outstanding bravery, compassion and t leading to ed
humanitarian engagement in rescuing dendritic resulting
civilians. pattern in in trellis,
Ref: plains radial and
https://www.thehindu.com/news/intern rectangul
ational/syrian-white-helmet-members- ar
flee-to-jordan/article24488994.ece patterns
https://www.theatlantic.com/news/arc 4. Nature of Long Smaller,
hive/2016/09/syria-white- River course, fixed
helmets/502073/ flowing course
through with well-
5. Which of the following are the the rugged adjusted
characteristic features of mountains valleys
Himalayan Rivers? experienci
(1) Super imposed, rejuvenated ng
drainage resulting in trellis, headward
radial and rectangular erosion
patterns. and river
(2) Old rivers with graded profile, capturing;
and have almost reached In plains
their base levels. meanderin
(3) Very large basins. g and
Select the correct answer using the shifting of
code given below: course.
(a) 1 and 2 only 5. Catchme Very large Relatively
(b) 2 and 3 only nt Area basins smaller
(c) 3 only basin
(d) 1, 2 and 3 6. Age of Young and Old rivers
the River youthful, with
Answer: (c) active and graded

3
Prelim IAS Test Series (2019) – GS Test 14 (17.03.2019)
GS (Paper-I) Full Test 1
Join Our Telegram Channel https://t.me/UPSCMaterials For Instant Updates

deepening profile, Source: Fundamentals of Physical


in the and have Geography, page: 26
valleys almost
reached 7. In March 1942, a Mission headed
their base by Stafford Cripps was sent to
levels India with constitutional proposals
Hence (c) is the correct answer. to seek Indian support for the war.
Source: INDIA: PHYSICAL Which of the following were the
ENVIRONMENT Page 29 main proposals of Cripps Mission?
(1) After the end of the war, a
6. Consider the following statements Constituent Assembly would
with reference to Volcanoes: be convened to frame a new
(1) Composite Volcanoes are Constitution and the making
the largest of all the of the Constitution was to be
volcanoes on the Earth. solely in Indian hands.
(2) Shield volcanoes are mostly (2) Option was to be made
made up of basalt and are available to any province to
not steep. have a separate Constitution.
(3) Caldera is the most explosive (3) Free India could withdraw
of the Earth’s volcanoes. from the Commonwealth.
Which of the statements given Select the correct answer using the
above are correct? code given below:
(a) 1 and 2 only (a) 1 and 2 only
(b) 1 and 3 only (b) 1 and 3 only
(c) 2 and 3 only (c) 2 and 3 only
(d) 1, 2 and 3 (d) 1, 2 and 3

Answer: (c) Answer: (d)


Explanation: Explanation:
Barring the basalt flows, the shield The Cripps proposals differed from those
volcanoes are the largest of all the offered in the past in many respects—
volcanoes on the Earth. Shield volcanoes • The making of the constitution was to
are mostly made up of basalt, a type of be solely in Indian hands now (and
lava that is very fluid when erupted. For not "mainly" in Indian hands - as
this reason, these volcanoes are not contained in the August Offer). A
steep (you can’t pile up a fluid that easily concrete plan was provided for the
runs downhill). They become explosive if Constituent Assembly. Hence,
somehow water gets into the vent; statement (1) is correct.
otherwise, they are characterised by low- • Option was to be made available to
explosivity. Hence, statement (1) is any province to have a separate
incorrect and statement (2) is correct. Constitution - a blueprint for India's
Caldera is the most explosive of the partition. Hence, statement (2) is
Earth’s volcanoes. They are usually so correct.
explosive that when they erupt they tend • Free India could withdraw from the
to collapse on themselves rather than Commonwealth. Hence, statement
building any tall structure. Hence, (3) is correct.
statement (3) is correct.

4
Prelim IAS Test Series (2019) – GS Test 14 (17.03.2019)
GS (Paper-I) Full Test 1
Join Our Telegram Channel https://t.me/UPSCMaterials For Instant Updates

• Indians were allowed a large share in (3) It recommended English as the


the administration in the interim medium of instruction for higher
period. studies and vernaculars at school
Source: Spectrum - A Brief History of level.
Modern India Page 207. (4) It laid stress on female and
vocational, education, and on
8. Charles Wood prepared a teachers' training.
despatch on an educational system (5) It laid down that the education
for India and it is considered as the imparted in government institutions
'Magna Carta' of English Education should be secular.
in India. With regard to Woods (6) It recommended a system of grants-
Despatch, consider the following in-aid to encourage private
statements: enterprise.
(1) It recommended English as Hence statements (1) and (2) are
the medium of instruction for correct.
higher studies and Transfer of control of primary education
vernaculars at school level. to newly set up district and municipal
(2) It recommended a system of boards is the recommendation of
grants-in-aid to encourage HUNTER EDUCATION COMMISSION
private enterprise. (1882-83). Hence, statement (3) is
(3) It recommended transfer of incorrect.
control of primary education
to newly set up district and 9. Which among the following could
municipal boards. be the reasons for the sluggish tax-
Which of the statements given to-GDP ratio in India?
above are correct? (1) Lower per capita income
(a) 1 and 2 only (2) Tax litigations
(b) 1 and 3 only (3) Tax exemptions for
(c) 2 and 3 only agricultural income
(d) 1, 2 and 3 (4) Reduction in the corporate
income tax from 30% to 25%
Answer: (a) Select the correct answer using the
Explanation: code given below:
Woods Despatch: (a) 1 and 2 only
(1) It asked the Government of India to (b) 1, 2 and 3 only
assume responsibility for education (c) 2, 3 and 4 only
of the masses, thus repudiating the (d) 1, 2, 3 and 4
‘downward filtration theory', at least
on paper. Answer: (b)
(2) It systematised the hierarchy from Explanation:
vernacular primary schools in villages Tax to gross domestic product (GDP)
at bottom, followed by Anglo- ratio is the ratio of taxes collected by a
Vernacular High Schools and an government and the GDP of the nation.
affiliated college at the district level, Taxes constitute an important component
and affiliating universities in the of revenue and the aforesaid ratio is a
presidency towns of Calcutta, key barometer that indicates the ability of
Bombay and Madras. the government to invest in various

5
Prelim IAS Test Series (2019) – GS Test 14 (17.03.2019)
GS (Paper-I) Full Test 1
Join Our Telegram Channel https://t.me/UPSCMaterials For Instant Updates

development initiatives. India has had a (d) 1, 2 and 3


comparatively low tax-to-GDP
ratio. Some of the reasons for this Answer: (b)
sluggish tax-to-GDP ratio could be Explanation:
attributed to factors such as the so-called The term ‘Repo’ stands for ‘Repurchase
parallel economy, lower per capita agreement’. Repo is form of short-term,
income, tax litigations, tax exemptions for interest-bearing and collateral-backed
agricultural income and lower borrowing. Repo is basically a short-term
compliances. Hence, statements (1), (2) money market instrument which is used
and (3) are correct. High corporate to raise capital for the shorter-term. In
income tax rates and a narrow base Indian Banking terms, repo rate is the
distort the allocation of resources, rate at which the Reserve Bank of India
discourage foreign investment and make lends money to all the Commercial
tax evasion and avoidance more Banks in the country in the event of
attractive. Therefore, Budget 2018 scarcity of funds. The reverse repo is the
reduced corporate tax rate from 30% to rate at which the central bank borrows
25% for companies with turnover of up to money from the Commercial Banks
Rs 250 crore with an aim of broadening within the country. It is a monetary policy
the tax base and improving the tax to instrument which can be used to control
GDP ratio. Hence, (b) is the correct the money supply in the country. An
answer. increase in reverse repo rate means that
Sources: the Commercial Banks will get more
https://www.livemint.com/Money/GIvXl incentives to park their funds with the
K7F1cpYwvj57MBD5H/Impact-of- RBI, thereby decreasing the supply of
better-taxtoGDP-ratio.html money in the market. This may cause
https://www.oecd.org/eco/surveys/IND the rupee to appreciate. Hence,
IA-2017-OECD-economic-survey- statements (1) and (2) are correct. Due
overview.pdf to reduced money supply in the market,
https://economictimes.indiatimes.com inflation will also come down. Increased
/small-biz/policy-trends/budget-2018- reverse repo rate helps common man by
corporate-tax-rate-reduced-to-25-for- curbing the rate of inflation. Hence (b) is
companies-with-turnover-of-up-to-rs- the correct answer.
250-crore/articleshow/62738932.cms Sources: Class XII: NCERT:
Introductory Macroeconomics:
10. Which among the following can be Chapter 3: Page No 51
the implications of increasing https://www.paisabazaar.com/rbi/repo-
reverse repo rate by the Reserve rate-and-reverse-repo-rate/
Bank of India?
(1) Appreciation of rupee 11. ‘Directive Principles of State Policy’
(2) Depreciation of rupee under Part IV of the Indian
(3) Reduced money supply in Constitution, are the ideals that the
the market State should keep in mind while
Select the correct answer using the formulating policies and enacting
code given below: laws. With reference to Directive
(a) 1 and 2 only Principles of State Policy, consider
(b) 1 and 3 only the following statements:
(c) 2 and 3 only

6
Prelim IAS Test Series (2019) – GS Test 14 (17.03.2019)
GS (Paper-I) Full Test 1
Join Our Telegram Channel https://t.me/UPSCMaterials For Instant Updates

(1) Directive Principles of State Which of the statements given


Policy aim at establishing above is/are correct?
political democracy in the (a) 1 only
country. (b) 2 only
(2) The courts can declare a law (c) Both 1 and 2
violative of any of the (d) Neither 1 nor 2
Directive Principles as
unconstitutional and invalid. Answer: (b)
Which of the statements given Explanation:
above is/are correct? The Constitution deals with citizenship in
(a) 1 only Articles 5 to 11 under Part II. However, it
(b) 2 only contains neither any permanent nor any
(c) Both 1 and 2 elaborate provisions in this regard. It only
(d) Neither 1 nor 2 identifies the persons who became
citizens of India at its commencement
Answer: (d) (i.e., on January 26, 1950). Hence,
Explanation: statement 1 is incorrect.
Directive Principles of State Policy aim at It does not deal with the problem of
establishing social and economic acquisition or loss of citizenship
democracy in the country. It is the subsequent to its commencement. It
Fundamental rights which aim at empowers the Parliament to enact a law
establishing political democracy in the to provide for such matters and any other
country. Hence, statement (1) is matter relating to citizenship. Hence,
incorrect. statement 2 is correct.
The courts cannot declare a law violative Source: Indian Polity- M Laxmikanth,
of any of the Directive Principles as 5th Edition chapter 6; Citizenship;
unconstitutional and invalid. However, page- 6.1
they can uphold the validity of a law on
the ground that it was enacted to give 13. Consider the following statements
effect to a directive. Hence, statement with regard to the National
(2) is incorrect. Pharmaceutical Pricing Authority
Source: Indian Polity- M Laxmikanth, (NPPA):
5th Edition ch-8; DPSP; page- 8.6 (1) Under the Drugs and
Cosmetics Act, 1940, NPPA
12. Consider the following statements is responsible for the
with reference to citizenship in the approval of new drugs,
Indian context: conduct of clinical trials,
(1) The Constitution of India laying down the standards for
contains permanent and drugs in the country.
elaborate provisions about (2) NPPA is also entrusted with
citizenship in India. the task of recovering
(2) The Constitution of India amounts overcharged by
does not deal with the manufacturers for controlled
problem of acquisition or loss drugs from the consumers.
of citizenship subsequent to Which of the statements given
its commencement. above is/are correct?
(a) 1 only

7
Prelim IAS Test Series (2019) – GS Test 14 (17.03.2019)
GS (Paper-I) Full Test 1
Join Our Telegram Channel https://t.me/UPSCMaterials For Instant Updates

(b) 2 only (c) Low weight for height among


(c) Both 1 and 2 children.
(d) Neither 1 nor 2 (d) Wastage of the demographic
dividend due to malnutrition.
Answer: (b)
Explanation: Answer: (c)
Under the Drugs and Cosmetics Act, Explanation:
1940, the Central Drugs Standard According to Global Nutrition Report
Control Organisation (CDSCO) (and 2018 published recently, India accounted
not NPPA) is responsible for approval of for 25.5 million children who are wasted,
new drugs, conduct of clinical trials, followed by Nigeria (3.4 million) and
laying down the standards for drugs, Indonesia (3.3 million). Also, India is
control over the quality of imported drugs facing a major malnutrition crisis as it
in the country. Hence, statement (1) is holds almost a third of the world’s burden
incorrect. for stunting.
NPPA is an organization of the • Wasting refers to low weight-for-height
Government of India which was where a child is thin for his/her height but
established, inter alia, to fix/revise the not necessarily short. Also known as
prices of controlled bulk drugs and acute malnutrition, this carries an
formulations and to enforce prices and immediate increased risk of morbidity
availability of the medicines in the and mortality. Wasted children have a 5-
country, under the Drugs (Prices Control) 20 times higher risk of dying from
Order, 1995. The organization is also common diseases like diarrhoea or
entrusted with the task of recovering pneumonia than normally nourished
amounts overcharged by manufacturers children.
for the controlled drugs from the • Stunting: Stunted growth refers to low
consumers. Hence, statement (2) is height-for-age, when a child is short for
correct. his/her age but not necessarily thin. Also
Sources: known as chronic malnutrition, this
https://www.thehindubusinessline.co carries long-term developmental risks.
m/economy/govt-to-regulate-four- Source:
more-medical- https://economictimes.indiatimes.com
devices/article25712030.ece /news/politics-and-nation/india-has-
http://www.nppaindia.nic.in/ one-third-of-worlds-stunted-children-
https://cdscoonline.gov.in/CDSCO/ho global-nutrition-
mepage report/articleshow/66865016.cms

14. Recently Global Nutrition Report 15. Which of the following comes
2018 has been released. In the under the organically formed
context of malnutrition, the term sedimentary rocks?
‘Wasting’ often seen in the news, (1) Sandstone
can be described as (2) Coal
(a) Low height for age among (3) Shale
children. (4) Geyserite
(b) Low mental growth among Select the correct answer using the
children due to malnutrition. code given below:
(a) 1, 2 and 3 only

8
Prelim IAS Test Series (2019) – GS Test 14 (17.03.2019)
GS (Paper-I) Full Test 1
Join Our Telegram Channel https://t.me/UPSCMaterials For Instant Updates

(b) 2 and 3 only In middle latitudes, most of diurnal (day


(c) 1, 2 and 4 only and night) variation in daily weather are
(d) 2 and 4 only caused by advection alone. Hence,
statement (2) is correct.
Answer: (d) In tropical regions, particularly in
Explanation: Northern India, during summer season
Depending upon the mode of formation, local winds called ‘loo’ are the outcome
sedimentary rocks are classified into of advection process. Hence, statement
three major groups: (3) is correct.
(i) Mechanically formed - sandstone, Source: Fundamentals of Physical
conglomerate, limestone, shale, Geography, page: 80-81
loess etc.
(ii) Organically formed - geyserite, 17. Which of the following were the
chalk, limestone, coal etc. issues behind Eka Movement?
(iii) Chemically formed - chert, (1) High rents - 50 per cent
limestone, halite, potash etc. higher than the recorded
Hence (d) is the correct answer. rates.
Source: Fundamentals of Physical (2) Oppression by thikadars who
Geography, page: 42 were in charge of revenue
collection.
16. Consider the following statements (3) Practice of share-rents.
with regard to Advection: Select the correct answer using the
(1) The transfer of heat through codes given below:
horizontal movement of air is (a) 1 and 2 only
called advection. (b) 1 and 3 only
(2) In middle latitudes, most of (c) 2 and 3 only
diurnal (day and night) (d) 1, 2 and 3
variation in daily weather are
caused by advection alone. Answer: (d)
(3) In Northern India during the Explanation:
summer season local winds Eka Movement: Towards the end of
called ‘loo’ are the outcome 1921, peasant discontent resurfaced in
of advection process. some northern districts of the United
Which of the statements given Provinces - Hardoi, Bahraich, Sitapur.
above is/are correct? The issues involved were:
(a) 1 only (i) high rents - 50 per cent higher than
(b) 1 and 2 only the recorded rates;
(c) 2 and 3 only (ii) oppression by thikadars who were in
(d) 1, 2 and 3 charge of revenue collection; and
(ii) practice of share-rents.
Answer: (d) The meetings of the Eka or the Unity
Explanation: Movement involved a symbolic religious
The process of vertical heating of the ritual in which the assembled peasants
atmosphere is known as convection and vowed that they would
the transfer of heat through horizontal • pay only the recorded rent but would
movement of air is called advection. pay it on time;
Hence, statement (1) is correct. • not leave when evicted;

9
Prelim IAS Test Series (2019) – GS Test 14 (17.03.2019)
GS (Paper-I) Full Test 1
Join Our Telegram Channel https://t.me/UPSCMaterials For Instant Updates

• refuse to do forced labour; on National Economic Programme—


• give no help to criminals; which made the session particularly
• abide by panchayat decisions. memorable.
The grassroot leadership of the Eka This was the first time the Congress spelt
Movement came from Madari Pasi and out what swaraj would mean for the
other low-caste leaders, and many small masses - "in order to end exploitation of
zamindars. By March 1922, severe masses, political freedom must include
repression by authorities brought the economic freedom of starving millions."
movement to an end. Hence, option (d) Hence (b) is the correct answer
is the correct answer. Source: Spectrum - A brief History of
Source: Spectrum - A brief History of Modern India Page 176
Modern India Page 337
19. ‘Inclusive Wealth Report, 2018’ is
18. Consider the following events prepared by the
during pre-independence India: (a) UN Environment Programme
(1) Endorsement of the Delhi Pact (b) International Monetary Fund
(Gandhi-Irwin Pact) (c) International Bank for
(2) Resolution on Fundamental Reconstruction and
Rights Development
(3) Spelling out the meaning of (d) Organization for Economic
swaraj Cooperation and
The events given above are Development
features of which of the following
sessions of the Indian National Answer: (a)
Congress? Explanation:
(a) Gaya session The UN Environment Programme
(b) Karachi session (UNEP, or UN Environment) and
(c) Lahore session partners released preliminary findings
(d) Haripura session from the ‘Inclusive Wealth Report 2018’
that aims to evaluate and report on a
Answer: (b) country’s wealth and well-being. The
Explanation: Inclusive Wealth Report is a biennial
In March 1931, a special session of the report that ranks 140 countries using the
Congress was held at Karachi to endorse Inclusive Wealth Index, a tool that
the Gandhi-Irwin or Delhi Pact. assesses a country’s ability to look after
Congress Resolutions at Karachi its wealth in a way that is sustainable and
• While disapproving of and safeguards its future generations. The
dissociating itself from political top ten performers on the basis of per
violence, the Congress admired the capita inclusive wealth for 1992-2014 are
"bravery" and "sacrifice" of the three the Republic of Korea, Singapore, Malta,
martyrs i.e. Bhagat Singh, Rajguru Latvia, Ireland, Moldova, Estonia,
and Sukhdev. Mauritius, Lithuania and Portugal
• The Delhi Pact was endorsed. Source: Vajiram and Ravi: Current
• The goal of Purna Swaraj was Affairs for November 2018: Page No
reiterated. 67
• Two resolutions were adopted - one
on Fundamental Rights and the other

10
Prelim IAS Test Series (2019) – GS Test 14 (17.03.2019)
GS (Paper-I) Full Test 1
Join Our Telegram Channel https://t.me/UPSCMaterials For Instant Updates

20. Consider the following statements The Scheme provides for an Appellate
with reference to the Ombudsman mechanism under which the complainant
scheme for NBFCs: NBFC has the option to appeal against
(1) The scheme will provide the decision of the Ombudsman before
cost-free and expeditious the Appellate Authority. The offices of the
complaint redressal NBFC ombudsmen will function at four
mechanism relating to metro centres; Chennai, Kolkata,
deficiency in services by non- Mumbai and New Delhi. Hence,
banking financial companies statement (3) is correct.
covered under the scheme. Value Addition:
(2) The Central government will The NBFC Ombudsman does not charge
appoint joint secretary level any fee for filing and resolving
officers to be known as customer’s complaints. It will be the
Ombudsman to carry out the obligation of NBFC concerned to
functions and the implement settlement arrived with
appointment will be for a complainant or Award passed by
period of not exceeding three Ombudsman when it becomes final and
years at a time. send report in this regard to the RBI
(3) The offices of the NBFC within 15 days of award becoming final.
ombudsmen will function at Source: Vajiram Indian Economy
four metro centres; Chennai, 2019: Chapter 4 Money and Banking:
Kolkata, Mumbai and New Page No 31-32
Delhi. https://economictimes.indiatimes.com
Which of the statements given /industry/banking/finance/rbi-
above is/are correct? launches-ombudsman-scheme-for-
(a) 1 and 2 only nbfcs/articleshow/63049092.cms
(b) 1 and 3 only
(c) 2 and 3 only 21. Consider the following statements
(d) 1, 2 and 3 with reference to the Pitt’s India Act
of 1784:
Answer: (b) (1) It prohibited the servants of
Explanation: the Company from engaging
The ombudsmen scheme for NBFCs will in any private trade or
provide cost-free and expeditious accepting presents or bribes
complaint redressal mechanism relating from the ‘natives’.
to deficiency in services by non-banking (2) It distinguished between the
financial companies covered under the commercial and political
scheme. Hence, statement (1) is functions of the Company.
correct. (3) The British Government was
The Reserve Bank may appoint one or given the supreme control
more of its officers in the rank of not less over Company’s affairs and
than General Manager to be known as its administration in India
Ombudsman to carry out the functions. Which of the statements given
The appointment of Ombudsman may be above are correct?
made for a period not exceeding three (a) 1 and 2 only
years at a time. Hence, statement (2) is (b) 1 and 3 only
incorrect. (c) 2 and 3 only

11
Prelim IAS Test Series (2019) – GS Test 14 (17.03.2019)
GS (Paper-I) Full Test 1
Join Our Telegram Channel https://t.me/UPSCMaterials For Instant Updates

(d) 1, 2 and 3 located within its territorial


jurisdiction only.
Answer: (c) (3) The writ of habeas corpus
Explanation: can be issued against both
The main features of Pitt’s India Act of public authorities as well as
1784 includes private individuals.
1. It distinguished between the Which of the statements given
commercial and political functions of above are correct?
the Company. (a) 1 and 2 only
2. It allowed the Court of Directors to (b) 1 and 3 only
manage the commercial affairs but (c) 2 and 3 only
created a new body called Board of (d) 3 only
Control to manage the political
affairs. Thus, it established a system Answer: (c)
of double government. Explanation:
3. It empowered the Board of Control to The Supreme Court can issue writs only
supervise and direct all operations of for the enforcement of fundamental rights
the civil and military government or whereas a High Court can issue writs not
revenues of the British possessions only for the enforcement of Fundamental
in India. Rights but also for any other purpose.
Thus, the act was significant for two The expression ‘for any other purpose’
reasons: first, the Company’s territories refers to the enforcement of an ordinary
in India were for the first time called the legal right. Thus, the writ jurisdiction of
‘British possessions in India’; and the Supreme Court, in this respect, is
second, the British Government was narrower than that of a High Court.
given the supreme control over Hence, statement (1) is incorrect.
Company’s affairs and its administration The Supreme Court can issue writs
in India. Hence, statements (2) and (3) against a person or government
are correct. throughout the territory of India whereas
Statement (1) is incorrect as it was the a high court can issue writs against a
feature of Regulating Act of 1773. person residing or against a government
Source: Indian Polity- M Laxmikanth, or authority located within its territorial
5th Edition ch-1; Historical jurisdiction only or outside its territorial
Background; page- 1.3 jurisdiction only if the cause of action
arises within its territorial jurisdiction.
22. Consider the following statements Hence, statement (2) is correct.
with reference to “writs”: The writ of habeas corpus can be issued
(1) The Supreme Court can against both public authorities as well as
issue writs for the private individuals. The writ, on the other
enforcement of fundamental hand, is not issued where the (a)
rights as well as for any other detention is lawful, (b) the proceeding is
purpose. for contempt of a legislature or a court,
(2) The High Court can issue (c) detention is by a competent court,
writs against a person and (d) detention is outside the
residing or against a jurisdiction of the court. Hence,
government or authority statement (3) is correct.

12
Prelim IAS Test Series (2019) – GS Test 14 (17.03.2019)
GS (Paper-I) Full Test 1
Join Our Telegram Channel https://t.me/UPSCMaterials For Instant Updates

Source - Indian Polity- M Laxmikanth, (1) It is the state animal of


4th Edition ch-7; Fundamental Rights; Manipur.
page- 7.18 (2) It enjoys the same level of
protection as that of Clouded
23. Consider the following statements Leopard under the Wildlife
with reference to the Bay of Bengal (Protection) Act, 1972.
Initiative for Multi-Sectoral Which of the statements given
Technical and Economic above is/are correct?
Cooperation (BIMSTEC): (a) 1 only
(1) BIMSTEC was established in (b) 2 only
1997 on the basis of the (c) Both 1 and 2
Bangkok Declaration. (d) Neither 1 nor 2
(2) The fourth BIMSTEC summit
was recently held in Goa. Answer: (c)
(3) It has a permanent Explanation:
Secretariat at Dhaka. Brow-antlered deer is the state animal of
Which of the statements given Manipur. Hence, statement (1) is
above is/are correct? correct. Both Brow-antlered deer and
(a) 1 only Clouded Leopard are given protection
(b) 1 and 2 only under Schedule-1 of the Wildlife
(c) 1 and 3 only (Protection) Act, 1972. Schedule I
(d) 1, 2 and 3 provides absolute protection and
offences under these are prescribed the
Answer: (c) highest penalties. Also Brow-antlered
Explanation: deer has Endangered status under the
BIMSTEC was established on 6th June IUCN Red list. Hence, statement (2) is
1997 through the Bangkok Declaration. correct.
Recently the fourth summit of BIMSTEC Recently, there was a tug of war between
was held in Kathmandu, Nepal and the State government, activists and local
decision has been taken on drafting a residents for finding a second home for
new charter for BIMSTEC. Hence, the deer.
statement (1) is correct and statement Source -
(2) is incorrect. https://www.thehindu.com/sci-
BIMSTEC Permanent Secretariat is at tech/energy-and-environment/in-
Dhaka which was opened in 2014 and search-of-a-second-home-for-
India provides 33% (65% of region's manipurs-brow-antlered-
population) of its expenditure. Hence, deer/article24843987.ece
statement (3) is correct.
Ref: 25. Consider the following statements
https://www.thehindu.com/opinion/op- with reference to the tributaries of
ed/green-shoots-of- Indus River System:
revival/article24857894.ece (1) Tributaries such as Shyok,
Gilgit join the left bank of the
24. Consider the following statements Indus.
with reference to Brow-antlered (2) The Jhelum, an important
deer which was in news recently: tributary of the Indus, rises
from west of the Rohtang

13
Prelim IAS Test Series (2019) – GS Test 14 (17.03.2019)
GS (Paper-I) Full Test 1
Join Our Telegram Channel https://t.me/UPSCMaterials For Instant Updates

pass in the Kullu hills of B. Banihal 2. Zaskar


Himachal Pradesh. C. Photu La 3. Pir Panjal
Which of the statements given D. Khardung 4. Great
above is/are correct? La Himalaya
(a) 1 only
(b) 2 only Codes
(c) Both 1 and 2 A B C D
(d) Neither 1 nor 2 (a) 4 3 2 1
(b) 1 2 3 4
Answer: (d) (c) 1 4 2 3
Explanation: (d) 2 3 1 4
Indus originates from a glacier near
Bokhar Chu in the Tibetan region at an Answer: (a)
altitude of 4,164 m in the Kailash Explanation:
Mountain range. After flowing in the Passes Mountain Range
northwest direction between the Ladakh Zoji La - Great Himalayas
and Zaskar ranges, it passes through Banihal - Pir Panjal
Ladakh and Baltistan. The Indus receives Photu La - Zaskar
a number of Himalayan tributaries such Khardung La - Ladakh range.
as the Shyok, the Gilgit, the Zaskar, the Source - INDIA : PHYSICAL
Hunza, the Nubra, the Shigar, the ENVIRONMENT Page 11
Gasting and the Dras which joins the
right bank of Indus. Hence, statement 27. Match the following pairs:
(1) is incorrect. Newspaper Journalist
The Jhelum, an important tributary of the A. Maharatta 1. G.
Indus, rises from a spring at Verinag Subramaniya
situated at the foot of the Pir Panjal in the Iyer
south-eastern part of the valley of B. Swadesamitran 2. Bal Gangadhar
Kashmir. It flows through Srinagar and Tilak
the Wular lake before entering Pakistan C. Voice of India 3. Gopal Krishna
through a deep narrow gorge. Hence, Gokhale
statement (2) is incorrect. D. Sudharak 4. Dadabhai
It is the ‘Ravi’, another important tributary Naoroji
of the Indus, which rises west of the Codes
Rohtang pass in the Kullu hills of A B C D
Himachal Pradesh and flows through the (a) 1 2 3 4
Chamba valley of the state. (b) 2 1 4 3
Source: INDIA: PHYSICAL (c) 2 3 4 1
ENVIRONMENT Page 25 (d) 2 1 3 4
26. Match the following passes with the Answer: (b)
mountain range they are situated Explanation:
in: Newspaper Journalist
Passes Mountain 1. Maharatta A. Balagangadhar Tilak
Range 2. Swadesamitran B. G. Subramaniya
A. Zoji La 1. Ladakh Aiyer
Range

14
Prelim IAS Test Series (2019) – GS Test 14 (17.03.2019)
GS (Paper-I) Full Test 1
Join Our Telegram Channel https://t.me/UPSCMaterials For Instant Updates

3. Voice of India C. Dadabhai Naoroji Answer: (b)


4. Sudharak D. Gopal Krishna Gokhale Explanation:
Additional information: C.R. Das moved the main resolution on
Many newspapers emerged under non-cooperation in the annual session of
distinguished and fearless journalists. the Congress in Nagpur in 1920 and
These included - The Hindu and played a major role in promoting the
Swadesamitran under G. Subramaniya movement. Hence, statement (1) is
Aiyar, The Bengalee under Surendranath correct.
Banerjee, Voice of India under Dadabhai Lokmanya Tilak spread the message of
Naoroji, Amrita Bazar Patrika under Sisir Swadeshi to Poona and Bombay and
Kumar Ghosh and Motilal Ghosh, Indian organised Ganapati and Shivaji festivals
Mirror under N.N. Sen, Kesari (in to arouse patriotic feelings. He stressed
Marathi) and Maharatta (in English) that the aim of swadeshi, boycott and
under Balgangadhar Tilak, Sudharak national education was attainment of
under Gopal Krishna Gokhale, and swaraj. The statement given was related
Hindustan and Advocate under G.P. to Swadeshi movement not Non-
Verma Cooperation Movement. Hence,
Source: Spectrum - A brief History of statement (2) is incorrect.
Modern India Page 285 Subhash Chandra Bose supported the
movement and resigned from the civil
28. M. K. Gandhi issued a manifesto in service. Hence, statement (3) is
March 1920, announcing his correct.
doctrine of non-violent Non- Source: Spectrum - A brief History of
Cooperation Movement. With Modern India Page 359.
reference to the Non-Cooperation
Movement consider the following 29. Consider the following statements
statements: with reference to inflation:
(1) C.R. Das moved the main (1) Reflation is a deliberate
resolution on non- policy adopted by the
cooperation at the annual government and monetary
session of the Congress in authorities to counter
Nagpur. deflationary situation.
(2) Lokmanya Tilak organised (2) Stagflation is a phenomenon
Ganapati and Shivaji which is characterized by the
festivals to arouse patriotic rise in price for a prolonged
feelings. period of time and relatively
(3) Subhash Chandra Bose low unemployment.
supported the movement and Which of the statements given
resigned from the civil above is/are correct?
service. (a) 1 only
Which of the statements given (b) 2 only
above are correct? (c) Both 1 and 2
(a) 1 and 2 only (d) Neither 1 nor 2
(b) 1 and 3 only
(c) 2 and 3 only Answer: (a)
(d) 1, 2 and 3 Explanation:

15
Prelim IAS Test Series (2019) – GS Test 14 (17.03.2019)
GS (Paper-I) Full Test 1
Join Our Telegram Channel https://t.me/UPSCMaterials For Instant Updates

Inflation means rising prices of goods Explanation:


and services. While this happens, prices The Exchange Traded Funds (ETFs) are
of some goods may be rising while those mutual funds listed and traded on stock
of other goods may not be rising. Inflation exchanges like shares. These mutual
manifests itself in various forms from funds primarily track the price of an asset
time to time depending upon causal like stock, gold etc. Hence, statement
factors, intensify and the kind of sectors it (1) is correct.
is related to. They can be bought and sold at any time
When prices are gently rising, it is and can be traded in secondary market.
referred as Creeping Inflation. In an ETF, one can buy and sell units at
Reflation is a deliberate policy adopted prevailing market price on a real time
by the government and monetary basis during market hours. The price of
authorities to counter deflationary an ETF’s shares will change throughout
situation. This is done by higher public the day as they are bought and sold. The
expenditure, tax cuts, interest rate largest ETFs typically have higher
reduction, etc. Hence, statement (1) is average daily volume and lower fees
correct. than mutual fund shares which makes
Stagflation is a typical situation in which them an attractive alternative for
inflation coexists with slowdown and individual investors. Hence, statement
unemployment. It is essentially a (2) is incorrect.
combination of high inflation and low Source - Vajiram: Indian Economy
growth. Chronic Inflation is a 2019: Chapter 10: Page No 117.
phenomenon which is characterized by https://www.thehindubusinessline.co
the rise in price for a prolonged period of m/opinion/columns/all-you-wanted-to-
time. Hence, statement (2) is incorrect. know-about-exchange-traded-
Source: Vajiram Indian Economy funds/article9957174.ece
2019: Chapter 5: Inflation: Page No 48-
49. 31. With reference to Preamble,
consider the following statements:
30. Consider the following statements (1) The Indian Constitution was
with reference to the Exchange the first to begin with a
Traded Funds (ETFs): Preamble.
(1) They are mutual funds listed (2) The Preamble to the Indian
and traded on stock Constitution is based on the
exchanges like shares. ‘Objectives Resolution’,
(2) They can be bought and sold drafted and moved by Motilal
at any time but cannot be Nehru.
traded in the secondary (3) The Preamble reveals the
market. source of authority of the
Which of the statements given Constitution.
above is/are correct? Which of the statements given
(a) 1 only above is/are correct?
(b) 2 only (a) 1 only
(c) Both 1 and 2 (b) 2 and 3 only
(d) Neither 1 nor 2 (c) 3 only
(d) 1, 2 and 3
Answer: (a)

16
Prelim IAS Test Series (2019) – GS Test 14 (17.03.2019)
GS (Paper-I) Full Test 1
Join Our Telegram Channel https://t.me/UPSCMaterials For Instant Updates

Answer: (c) Which of the statements given


Explanation: above is/are correct?
The American Constitution was the first (a) 1 only
to begin with a Preamble. Hence, (b) 2 only
statement (1) is incorrect. (c) 1 and 3 only
The Preamble to the Indian Constitution (d) 2 and 3 only
is based on the ‘Objectives Resolution’,
drafted and moved by Pandit Jawaharlal Answer: (a)
Nehru, and adopted by the Constituent Explanation:
Assembly. Hence, statement (2) is The Speaker of the Lok Sabha is
incorrect. provided with a security of tenure. He
The Preamble reveals four ingredients or can be removed only by a resolution
components: passed by the Lok Sabha by an absolute
1. Source of authority of the majority (ie, a majority of the total
Constitution: The Preamble states members of the House) and not by an
that the Constitution derives its ordinary majority (ie, a majority of the
authority from the people of India. members present and voting in the
2. Nature of Indian State: It declares House). Hence, statement (1) is
India to be a sovereign, socialist, correct.
secular democratic and republican The Deputy Speaker has one special
polity. privilege, that is, whenever he is
3. Objectives of the Constitution: It appointed as a member of a
specifies justice, liberty, equality and parliamentary committee, he
fraternity as the objectives. automatically becomes its Chairman. The
4. Date of adoption of the Constitution: Speaker appoints the Chairman of all the
It stipulates November 26, 1949 as parliamentary committees of the Lok
the date. Sabha and supervises their functioning.
Hence, statement (3) is correct. He himself of the Chairman of the
Source: Indian Polity - M Laxmikanth, Business Advisory Committee, the Rules
4th Edition ch-4; Preamble of the Committee and the General Purpose
Constitution; page- 4.1 Committee. Hence, statement (2) is
incorrect.
32. With reference to the Speaker of The Speaker of the Lok Sabha derives
Lok Sabha, consider the following his powers and duties from three
statements: sources, that is, the Constitution of India,
(1) The Speaker of Lok Sabha is the Rules of Procedure and Conduct of
provided with a security of Business of Lok Sabha, and
tenure. Parliamentary Conventions (residuary
(2) Whenever the Speaker is powers that are unwritten or unspecified
appointed as the member of in the Rules). Hence, statement (3) is
a parliamentary committee, incorrect.
he automatically becomes its Source - Indian Polity- M Laxmikanth,
Chairman. 4th Edition ch-22; Parliament; page-
(3) The Speaker of the Lok 22.7
Sabha derives his powers
and duties from Constitution
of India only.

17
Prelim IAS Test Series (2019) – GS Test 14 (17.03.2019)
GS (Paper-I) Full Test 1
Join Our Telegram Channel https://t.me/UPSCMaterials For Instant Updates

33. Consider the following statements 3. Sagar : Odisha


with reference to S-400 Triumf Air 4. Smith : Andaman
Defence System: Select the correct answer using the
(1) It is an air defence missile code given below:
system developed by Russia. (a) 1 and 3 only
(2) India will be the first foreign (b) 1 and 2 only
country to seal a government (c) 1, 2 and 4 only
- to - government deal to (d) 1, 3 and 4 only
procure this lethal missile
system. Answer: (c)
Which of the statements given Explanation:
above is/are correct? Ghoramara - West Bengal
(a) 1 only Minicoy - Lakshadweep
(b) 2 only Sagar - West Bengal
(c) Both 1 and 2 Smith - Andaman
(d) Neither 1 nor 2 Recently, a report released by the
National Centre for Coastal Research
Answer: (a) (NCCR), under the Ministry of Earth
Explanation: Sciences, said West Bengal recorded the
The S-400 Triumph is an air defence maximum erosion of 63%, followed by
missile system developed by Almaz Puducherry 57%, Kerala 45%, and Tamil
Central Design Bureau of Russia. The Nadu 41%. Sagar, the biggest island of
new system replaced the S-300P and S- the Sunderbans archipelago is facing
200 air defence systems of the Russian severe erosion on its western and south
Army. The Russian-built S-400 Triumf - eastern parts. Islands like Ghoramara
identified by NATO as the SA-21 Growler and Mousuni are also facing erosion. The
- is the world’s most dangerous same trend has been observed at
operationally deployed modern long- Jambudweep Island and Henry Island in
range surface-to-air missile system. the Sunderbans.
Hence, statement (1) is correct. NITI Aayog has begun the preparatory
China was the first foreign country to seal process for a Concept Development Plan
a government-to-government deal with and detailed Master Plan for the holistic
Russia to procure this lethal missile development of four islands (Ross &
system in 2014. India is yet to procure S- Smith Island, Long Island, Avis Island
400 Triumf missile system from Russia. and Little Andaman) in Andaman &
Hence statement (2) is incorrect. Nicobar Islands
Source: Vajiram and Ravi current affairs Source:
magazine October https://www.thehindu.com/sci-
https://www.thehindu.com/opinion/edi tech/energy-and-environment/west-
torial/22-on-india-us-defence- bengal-where-erosion-leads-to-land-
relationship/article24896848.ece loss/article24842375.ece

34. Which among the following pairs 35. Consider the following statements
are correctly matched? with reference to Plate Boundaries:
Island State/UT (1) New crust is generated in
1. Ghoramara : West Bengal Divergent Boundaries.
2. Minicoy : Lakshadweep

18
Prelim IAS Test Series (2019) – GS Test 14 (17.03.2019)
GS (Paper-I) Full Test 1
Join Our Telegram Channel https://t.me/UPSCMaterials For Instant Updates

(2) In Convergent Boundaries, (4) ‘Kayals’ are a distinguishing


the crust is destroyed as one feature of the Eastern coastal
plate dived under another. plain.
(3) Mid-Atlantic Ridge is an Which of the statements given
example of Convergent above are correct?
Boundaries. (a) 1 and 2 only
Which of the statements given (b) 1 and 3 only
above is/are correct? (c) 2 and 3 only
(a) 1 and 2 only (d) 1, 3 and 4 only
(b) 1 and 3 only
(c) 2 only Answer: (a)
(d) 1, 2 and 3 Explanation:
The western coastal plains are an
Answer: (a) example of submerged coastal plain.
Explanation: Because of this submergence it is a
In Divergent Boundaries, new crust is narrow belt and provides natural
generated as the plates pull away from conditions for the development of ports
each other. The sites where the plates and harbours. Kandla, Mazagaon, JLN
move away from each other are called port Navha Sheva, Marmagao,
spreading sites. Hence, statement (1) is Mangalore, Cochin, etc. are some of the
correct. important natural ports located along the
In Convergent Boundaries, the crust is west coast. Eastern coastal plain is
destroyed as one plate dives under broader and is an example of an
another. The location where sinking of a emergent coast. Because of its emergent
plate occurs is called a subduction zone. nature, it has less number of ports and
Hence statement (2) is correct. harbours. The continental shelf extends
The Mid-Atlantic Ridge is the best-known up to 500 km into the sea, which makes it
example of divergent boundaries. At this, difficult for the development of good ports
the American Plate(s) is/are separated and harbours. Hence, statement (1) is
from the Eurasian and African Plates. correct and statement (3) is incorrect.
Hence, statement (3) is incorrect. The western coastal plains are narrow in
Source: Fundamentals of Physical the middle and get broader towards north
Geography, page: 36 and south. The rivers flowing through this
coastal plain do not form any delta.
36. Consider the following statements Hence, statement (2) is correct.
with reference to Coastal Plains: The Malabar coast which is in the
(1) The western coastal plains western coastal plain has got certain
are an example of distinguishing features in the form of
submerged coastal plain. ‘Kayals’ (backwaters), which are used for
(2) The rivers flowing through fishing, inland navigation and also due to
western coastal plains do not its special attraction for tourists. Hence,
form any delta. statement (4) is incorrect.
(3) The Eastern coastal plain is a Source - India: Physical Environment,
narrow belt and provides Page 18
natural conditions for the
development of ports and 37. With reference to Neolithic culture,
harbours. consider the following statements:

19
Prelim IAS Test Series (2019) – GS Test 14 (17.03.2019)
GS (Paper-I) Full Test 1
Join Our Telegram Channel https://t.me/UPSCMaterials For Instant Updates

(1) Practice of agriculture started (1) The Harappans did not


during this period. believe in ghosts and
(2) This is the period when evil forces.
copper and bronze came to (2) Linga worship was prevalent
be used. in Harappan culture.
(3) The Harappan culture is (3) Trees and animals were
considered as a part of worshipped by the
Neolithic culture. Harappans.
Which of the statements given Which of the statements given
above is/are correct? above is/are correct?
(a) 1 only (a) 1 only
(b) 2 only (b) 1 and 2 only
(c) 2 and 3 only (c) 2 and 3 only
(d) 1, 2 and 3 (d) 1, 2 and 3

Answer: (a) Answer: (c)


Explanation: Explanation:
A remarkable progress is noticed in Religious beliefs of Harappan culture
human civilization in the Neolithic Age. It From the seals, terracotta figurines and
is approximately dated from 6000 B.C to copper tablets we get an idea of the
4000 B.C. Neolithic remains are found in religious life of the Harappans. The main
various parts of India. The chief features are:
characteristic features of the Neolithic • The chief male deity was Pasupati,
culture are - the practice of agriculture, (proto-Siva)
domestication of animals, polishing of • The chief female deity was the
stone tools and the manufacture of Mother Goddess represented in
pottery. In fact, the cultivation of plants terracotta figurines.
and domestication of animals led to the • In latter times, Linga worship was
emergence of village communities based prevalent.
on sedentary life. Hence, statement (1) • Trees and animals were also
is correct. worshipped by Harappans. Hence,
The Neolithic period is followed by statements (2) and (3) are correct.
Chalcolithic (copper-stone) period when • They believed in ghosts and evil
copper and bronze came to be used. forces and used amulets as
Generally, Chalcolithic cultures had protection against them. Hence,
grown in river valleys. Most importantly, statement (1) is incorrect.
the Harappan culture is considered as a Source: Tamil Nadu Standard XI –
part of Chalcolithic culture. Hence 2nd History (Lesson 2 Pre-Historic
and 3rd statements are incorrect. India and the Harappan Culture page:
Source - Tamil Nadu Standard XI – 23)
History (Lesson 2 Pre-Historic
India And The Harappan Culture page: 39. Consider the following statements
15) with reference to the Market
Stabilization scheme (MSS):
38. Consider the following statements (1) It is a monetary policy
about the religious beliefs of intervention by the RBI to
the Harappan culture: withdraw excess liquidity by

20
Prelim IAS Test Series (2019) – GS Test 14 (17.03.2019)
GS (Paper-I) Full Test 1
Join Our Telegram Channel https://t.me/UPSCMaterials For Instant Updates

selling government securities (b) An investor who provides


in the economy. funds to replacement and
(2) The amount raised under the renovation of industries.
MSS gets credited to the (c) Individual investors who are
Government Account and is willing to buy government
utilised to fund its securities in the money
expenditures. market.
Which of the statements given (d) An investor providing start-up
above is/are correct? capital, usually for the long-
(a) 1 only term, to new entrepreneurs.
(b) 2 only
(c) Both 1 and 2 Answer: (d)
(d) Neither 1 nor 2 Explanation:
Angel investors invest in
Answer: (a) small startups or entrepreneurs. Often,
Explanation: angel investors are among an
Market Stabilization scheme (MSS) is a entrepreneur's family and friends. The
monetary policy intervention by the RBI capital angel investors provide may be a
to withdraw excess liquidity (or money one-time investment to help the business
supply) by selling government securities propel or an ongoing injection of money
in the economy. Hence, statement (1) is to support and carry the company
correct. The amount raised under the through its difficult early stages. Angel
MSS does not get credited to the investors are also called informal
Government Account but is maintained in investors, angel funders, private
a separate cash account with the RBI investors, seed investors or business
and are used only for the purpose of angels. These are affluent individuals
redemption/buy back of Treasury- who inject capital for startups in
Bills/Dated Securities issued under the exchange for ownership equity or
scheme. Hence, statement (2) is convertible debt. Some angel investors
incorrect. The Reserve Bank of India invest through crowd funding platforms
initiated the MSS scheme in 2004. To online or build angel investor networks to
control the surge of US dollars in the pool in capital.
Indian market, the RBI started buying US Source -
dollars while pumping in rupee. This https://www.investopedia.com/terms/a
eventually led to over-supply of the /angelinvestor.asp
domestic currency raising inflationary
expectations. MSS was introduced to 41. Consider the following statements
mop up this excess liquidity. MSS bonds with reference to the Financial Bills
have a fixed tenure and earn returns. in India:
Ref: (1) It can be introduced only in
http://www.arthapedia.in/index.php?tit the Lok Sabha and not in the
le=Market_Stabilization_Scheme_(MS Rajya Sabha.
S) (2) It cannot be rejected or
amended by the Rajya
40. The term angel investor means? Sabha.
(a) An investor providing short- (3) In case of a disagreement
term capital to industries. between the two Houses

21
Prelim IAS Test Series (2019) – GS Test 14 (17.03.2019)
GS (Paper-I) Full Test 1
Join Our Telegram Channel https://t.me/UPSCMaterials For Instant Updates

over such a Bill, the (1) The Supreme Court of India


President can summon a is authorised to grant, at its
joint sitting of the two Houses discretion, special leave to
to resolve the deadlock. appeal from any judgement
Which of the statements given passed by any court or
above are correct? tribunal in the country.
(a) 1 and 2 only (2) Special Leave may be
(b) 1 and 3 only related to any matter -
(c) 2 and 3 only constitutional, civil, criminal,
(d) 1, 2 and 3 income tax, labour, revenue,
advocates, etc.
Answer: (b) Which of the statements given
Explanation: above is/are correct?
Under Article 117(1), a Financial Bill is a (a) 1 only
Bill that contains not only any or all the (b) 2 only
matters mentioned in Article 110, but (c) Both 1 and 2
also other matters of general legislation. (d) Neither 1 nor 2
It can be introduced only in the Lok
Sabha and not in the Rajya Sabha, and it Answer: (b)
can be introduced only on the Explanation:
recommendation of the President. The Supreme Court is authorised to
Hence, statement (1) is correct. grant in its discretion special leave to
Financial Bills can be either rejected or appeal from any judgement in any matter
amended by the Rajya Sabha (except passed by any court or tribunal in the
that an amendment other than for country except military tribunal and court
reduction or abolition of a tax cannot be martial. Hence, statement (1) is
moved in either House without the incorrect.
recommendation of the President). Appeal by Special Leave contains the
Hence, statement (2) is incorrect. four aspects as under:
In case of a disagreement between the (i) It is a discretionary power and hence,
two Houses over such a bill, the cannot be claimed as a matter of right.
President can summon a joint sitting of (ii) It can be granted in any judgement
the two Houses to resolve the deadlock. whether final or interlocutory.
When the Bill is presented to the (iii) It may be related to any matter -
President, he can either give his assent constitutional, civil, criminal, income tax,
to the Bill or withhold his assent to the labour, revenue, advocates, etc.
Bill or return the Bill for reconsideration of (iv) It can be granted against any court or
both the Houses. Hence, statement (3) tribunal and not necessarily against a
is correct. high court (except a military court).
Source - Indian Polity- M Laxmikanth, Hence, statement (2) is correct.
4th Edition ch-22; Parliament; page- Source - Indian Polity- M Laxmikanth,
22.20 4th Edition ch-25; Supreme Court;
page- 25.6
42. With reference to Appeal by
Special Leave, consider the 43. ‘Druzhba-III’ exercise, recently in
following statements: news, was held between which of
the following countries?

22
Prelim IAS Test Series (2019) – GS Test 14 (17.03.2019)
GS (Paper-I) Full Test 1
Join Our Telegram Channel https://t.me/UPSCMaterials For Instant Updates

(a) US and UAE Recently Bihar’s Shahi Litchi and


(b) India and Sri Lanka Alphonso Mango of Maharashtra have
(c) Pakistan and Russia been registered as Geographical
(d) Indian, Brazil and South Indications (GI). A Geographical
Africa Indication or a GI is an indication used on
products that have a specific
Answer: (c) geographical origin and possess qualities
Explanation: or a reputation that is/are due to that
Druzhba is a military exercise between origin. It conveys an assurance of quality
Russia and Pakistan which was started and distinctiveness which is essentially
in 2016. The 3rd edition of the Pakistan- attributable to its origin in that defined
Russia Joint Training Exercise ‘Druzhba- geographical locality. The recently
III’ was held at the National Counter- launched tagline for GI in India is
Terrorism Centre in Pabbi, a Tehsil of Invaluable Treasures of Incredible India.
Khyber Pakhtunkhwa’s Nowshera District Hence, statement (1) is correct.
in 2018. Druzhba is a Russian word At the international level, it is governed
which means Friendship. Hence (c) is by WTO’s Agreement on Trade-Related
the correct answer. Aspects of Intellectual Property Rights
Source - Vajiram and Ravi current (TRIPS). Hence, statement (2) is
affairs magazine November correct.
Geographical Indications are typically
44. Consider the following statements used for agricultural products, foodstuffs,
with reference to Geographical wine and spirit drinks, handicrafts, and
Indication (GI): industrial products. Hence, statement
(1) It is used for products that (3) is incorrect.
have a specific geographical Source - Vajiram and Ravi current
origin and possess qualities affairs magazine October +
or a reputation that are due https://www.wipo.int/geo_indications/
to that origin. en/faq_geographicalindications.html
(2) At the international level, it is
governed by WTO’s 45. Consider the following statements
Agreement on Trade-Related with reference to the natural
Aspects of Intellectual vegetation in India:
Property Rights (TRIPS). (1) The Himalayan ranges show
(3) Geographical Indications are a succession of vegetation
typically used for agricultural from the tropical to the
products, foodstuffs and tundra.
handicrafts only. (2) Deciduous forests are found
Which of the statements given in the foothills of the
above is/are correct? Himalayas.
(a) 1 only (3) Lakshadweep has zero per
(b) 1 and 2 only cent forest area.
(c) 2 and 3 only Which of the statements given
(d) 1, 2 and 3 above are correct?
(a) 1 and 2 only
Answer: (b) (b) 1 and 3 only
Explanation: (c) 2 and 3 only

23
Prelim IAS Test Series (2019) – GS Test 14 (17.03.2019)
GS (Paper-I) Full Test 1
Join Our Telegram Channel https://t.me/UPSCMaterials For Instant Updates

(d) 1, 2 and 3 Answer: (d)


Explanation:
Answer: (d) Classification of Himalayas Special
Explanation: Feature
The Himalayan ranges show a 1. Himachal Himalayas: Dun formations
succession of vegetation from the 2. Kashmir Himalayas: Karewa
tropical to the tundra, which changes with formations
the altitude. Hence, statement (1) is 3. Sikkim Himalayas: Duar formations
correct. 4. Arunachal Himalayas: Fast flowing
Deciduous forests are found in the rivers
foothills of the Himalayas. It is succeeded The Himachal Himalayas: The two
by the wet temperate type of forests distinguishing features of this region from
between an altitude of 1,000-2,000 m. the point of view of physiography are the
Hence, statement (2) is correct. ‘Shiwalik’ and ‘Dun formations’. Some
The forest area is the area notified and important duns located in this region are
recorded as the forest land irrespective the Chandigarh-Kalka dun, Nalagarh
of the existence of trees, while the actual dun, Dehra Dun, Harike dun and the
forest cover is the area occupied by Kota dun, etc. Dehra Dun is the largest of
forests with canopy. The former is based all the duns with an approximate length
on the records of the State Revenue of 35-45 km and a width of 22-25 km.
Department, while the latter is based on Kashmir Himalayas: The Kashmir
aerial photographs and satellite Himalayas are famous for Karewa
imageries. And Lakshadweep has zero formations, which are useful for the
per cent forest area. Hence, statement cultivation of Zafran, a local variety of
(3) is correct. saffron. Karewas are the thick deposits of
Source - India : Physical Environment, glacial clay and other materials
Page 60 embedded with moraines.
Sikkim Himalayas: As compared to the
46. Consider the following pairs: other sections of the Himalayas, these
Classification Special along with the Arunachal Himalayas are
of Himalayas Feature conspicuous by the absence of the
1. Sikkim : Duar Shiwalik formations. In place of the
Himalayas formations Shiwaliks here, the ‘duar formations’ are
2. Arunachal : Dun important, which have also been used for
Himalayas formations the development of tea gardens.
3. Himachal : Fast The Arunachal Himalayas: These
Himalayas flowing extend from the east of the Bhutan
rivers Himalayas up to the Diphu pass in the
4. Kashmir : Karewa east. The general direction of the
Himalayas formation mountain range is from southwest to
Which of the pairs given above are northeast. Some of the important
correctly matched? mountain peaks of the region are Kangtu
(a) 1 and 2 only and Namcha Barwa. These ranges are
(b) 2 and 3 only dissected by fast-flowing rivers from the
(c) 3 and 4 only north to the south, forming deep gorges.
(d) 1 and 4 only Bhramaputra flows through a deep gorge
after crossing Namcha Barwa.

24
Prelim IAS Test Series (2019) – GS Test 14 (17.03.2019)
GS (Paper-I) Full Test 1
Join Our Telegram Channel https://t.me/UPSCMaterials For Instant Updates

Source - India: Physical Environment, Source: An introduction to Indian Art,


Page 11 Class XI Page: 127

47. Consider the following pairs: 48. With reference to Paintings in


Architectural Special Upper Palaeolithic period, consider
terms Features the following statements:
1. Harmika : Rectangular (1) In India, the earliest paintings
or square have been reported from the
hall in front Upper Palaeolithic period.
of sanctuary (2) The paintings of the Upper
or main Palaeolithic period are linear
temple representations.
2. Chaitya : Place of Which of the statements given
congregation above is/are correct?
and worship (a) 1 only
3. Panchayatan : Main temple (b) 2 only
surrounded (c) Both 1 and 2
by four sub- (d) Neither 1 nor 2
shrines in
each corner Answer: (c)
4. Jagmohan : Small Explanation:
square Prehistoric paintings have been found in
fencing many parts of the world. We do not really
about the know if Lower Palaeolithic people ever
anda or produced any art objects. But by the
semi-circular Upper Palaeolithic times we see a
dome of proliferation of artistic activities. In India
Stupa the earliest paintings have been reported
Which of the pairs given above from the Upper Palaeolithic times.
is/are correctly matched? Hence, statement (1) is correct.
(a) 1 and 2 only The paintings of the Upper Palaeolithic
(b) 2 and 3 only phase are linear representations, in
(c) 1 and 4 only green and dark red, of huge animal
(d) 1, 3 and 4 only figures, such as bisons, elephants, tigers,
rhinos and boars besides stick-like
Answer: (b) human figures. Hence, statement (2) is
Explanation: correct.
Term Description Source - An introduction to Indian Art,
1. Harmika: Small square fencing about Class XI Page: 1 and 3
the anda or semi-circular dome of Stupa
2. Chaitya: Place of congregation and 49. Consider the following statements
worship with reference to the Unified
3. Panchayatan: Main temple surrounded Payment Interface (UPI):
by four sub-shrines in each corner (1) It is a single-window mobile
4 Jagmohan: Rectangular or square hall payment system developed
in front of sanctuary or main temple by the National Payments
Corporation of India (NPCI).

25
Prelim IAS Test Series (2019) – GS Test 14 (17.03.2019)
GS (Paper-I) Full Test 1
Join Our Telegram Channel https://t.me/UPSCMaterials For Instant Updates

(2) It is built over the IMPS conomy/What-is-Unified-Payment-


infrastructure and allows a Interface/article14593189.ece
customer to pay directly from https://www.investopedia.com/terms/u
a bank account to different /unified-payment-interface-upi.asp
merchants, both online and
offline 50. Which of the following categories
(3) Per transaction limit is Rs. 1 will be entertained as Public
lakh and maximum number Interest Litigation?
of transactions is 20 in 24 (1) Petitions for early hearing of
hours. cases pending in High Courts
Which of the statements given and Subordinate Courts.
above is/are incorrect? (2) Admission to medical and
(a) 1 and 3 only other educational institutions.
(b) 2 and 3 only (3) Petitions pertaining to
(c) 3 only environmental pollution,
(d) 1, 2 and 3 disturbance of ecological
balance.
Answer: (c) Select the correct answer using the
Explanation: code given below:
The Unified Payment Interface is a real- (a) 1 and 3 only
time payment system, developed by (b) 2 and 3 only
National Payments Corporation of India (c) 3 only
facilitating inter-bank transactions. It is (d) 1, 2 and 3
designed to enable peer-to-peer inter-
bank transfers through a single two-click Answer: (c)
factor authentication process. The Explanation:
interface is regulated by the Reserve The cases falling under the following
Bank of India (RBI), India's central bank. categories will not be entertained as
UPI allows a customer to pay directly PIL:
from a bank account to different 1. Landlord–tenant matters
merchants, both online and offline, 2. Service matter and those pertaining
without the hassle of typing credit card to pension and gratuity
details, IFSC code, or net banking/wallet 3. Complaints against Central/ State
passwords. The system is said to be a Government departments and Local
safe and secure method of transferring Bodies except those 10 specific
money between two parties, and cuts out cases allowed under PIL.
the need to transact with physical cash or 4. Admission to medical and other
through a bank. Hence statement (2) is educational institutions
correct. The per transaction limit is Rs. 1 5. Petitions for early hearing of cases
lakh and maximum number of pending in High Courts and
transactions is 10 in 24 hours. Earlier, Subordinate Courts
the limit was 20 transactions per day, but Hence statements (1) and (2) are
after the revision, it has now been incorrect.
capped at 10. Hence, statement (3) is Petitions pertaining to environmental
incorrect. pollution, disturbance of ecological
Source: balance, drugs, food adulteration,
https://www.thehindu.com/business/E maintenance of heritage and culture,

26
Prelim IAS Test Series (2019) – GS Test 14 (17.03.2019)
GS (Paper-I) Full Test 1
Join Our Telegram Channel https://t.me/UPSCMaterials For Instant Updates

antiques, forest and wildlife and other through the Tropic of Cancer and drains
matters of public importance. Hence, into Arabian sea in Gujarat. Hence,
statement (3) is correct. statement (3) is correct.
Source - Ref: Indian Polity- M Source - INDIA: PHYSICAL
Laxmikanth, 5th Edition ch-29, Public ENVIRONMENT Page 28
Interest Litigation https://guj-
nwrws.gujarat.gov.in/showpage.aspx?
51. Consider the following statements contentid=1486&lang=english
with regard to rivers in India:
(1) Narmada flows between 52. In the context of minor relief
Satpura and Vindhya ranges. features of Ocean floor, “Atoll”
(2) Luni is the largest river refers to
system of Gujarat. (a) It is a mountain with pointed
(3) Mahi river flows cutting the summits, rising from the
Tropic of Cancer twice. seafloor that does not reach
Which of the statements given the surface of the ocean.
above is/are correct? (b) These are deep valleys,
(a) 1 and 2 only sometimes found cutting
(b) 3 only across the continental
(c) 1 and 3 only shelves and slopes, often
(d) 1, 2 and 3 extending from the mouths of
large rivers.
Answer: (c) (c) It is a flat-topped seamount.
Explanation: (d) These are low islands found
The Narmada originates on the western in the tropical oceans
flank of the Amarkantak plateau at a consisting of coral reefs
height of about 1,057 m. Flowing in a rift surrounding a central
valley between the Satpura in the south depression.
and the Vindhyan range in the north, it
forms a picturesque gorge in marble Answer: (d)
rocks and Dhuandhar waterfall near Explanation:
Jabalpur. Hence, statement (1) is Atoll: These are low islands found in the
correct. tropical oceans consisting of coral reefs
Luni is the largest river system of surrounding a central depression. It may
Rajasthan, west of Aravali. It originates be a part of the sea (lagoon), or
near Pushkar in two branches, i.e. the sometimes form enclosing a body of
Saraswati and the Sabarmati, which join fresh, brackish, or highly saline water.
with each other at Govindgarh. From Hence (d) is the correct option.
here, the river comes out of Aravali and Additional Information:
is known as Luni. It flows towards the Minor relief features of Ocean floor
west till Telwara and then takes a Mid-Oceanic Ridges: A mid-oceanic
southwest direction to join the Rann of ridge is composed of two chains of
Kuchchh. Hence, statement (2) is mountains separated by a large
incorrect. depression.
Mahi River originates from Vindhyachal Seamount: It is a mountain with pointed
Hills, Madhya Pradesh and meets in Bay summits, rising from the seafloor that
of Khambhat. It flows cutting twice does not reach the surface of the ocean.

27
Prelim IAS Test Series (2019) – GS Test 14 (17.03.2019)
GS (Paper-I) Full Test 1
Join Our Telegram Channel https://t.me/UPSCMaterials For Instant Updates

Seamounts are volcanic in origin. The canvas, and is manifested by rich


Emperor seamount, an extension of the colourful application, creative motifs and
Hawaiian Islands in the Pacific Ocean, is designs, and portrayal of simple themes,
a good example. mostly mythological in depiction.
Submarine Canyons: These are deep Some of the popular themes represented
valleys, some comparable to the Grand through this art form are
Canyon of the Colorado river. They are Thia Badhia - depiction of the temple of
sometimes found cutting across the Jagannath;
continental shelves and slopes, often Krishna Lila - enactment of Jagannath as
extending from the mouths of large Lord Krishna displaying his powers as a
rivers. The Hudson Canyon is the best child;
known submarine canyon in the world. Dasabatara Patti - the ten incarnations of
Guyots: It is a flat topped seamount. Lord Vishnu;
They show evidences of gradual Panchamukhi - depiction of Lord Ganesh
subsidence through stages to become as a five-headed deity.
flat topped submerged mountains. Hence, statement (2) is incorrect.
Source: Fundamentals of Physical Source -
Geography, page: 115 https://archive.india.gov.in/knowindia/
culture_heritage.php?id=106
53. Consider the following statements
with reference to the Pattachitra 54. Which of the following pairs is/are
paintings: correctly matched?
(1) Pattachitra style of painting is Term Meaning
one of the oldest and most 1. Chauth : 9-10 per cent
popular art forms of Andhra of the land
Pradesh. revenue paid
(2) These paintings do not depict to the head
mythological characters or collector in
images of deities, but depict the Deccan.
social life. 2. Sardeshmukhi : 25 Percent of
Which of the statements given the land
above is/are correct? revenue
(a) 1 only claimed by
(b) 2 only zamindars.
(c) Both 1 and 2 3. Khanqahs : House of rest
(d) Neither 1 nor 2 for travellers,
especially
Answer: (d) one kept by a
Explanation: religious
The Pattachitra style of painting is one of order.
the oldest and most popular art forms of Select the correct answer using the
Odisha. Hence, statement (1) is code given below:
incorrect. (a) 1 only
The name Pattachitra has evolved from (b) 3 only
the Sanskrit words patta, meaning (c) 1 and 3 only
canvas, and chitra, meaning picture. (d) 1, 2 and 3
Pattachitra is thus a painting done on

28
Prelim IAS Test Series (2019) – GS Test 14 (17.03.2019)
GS (Paper-I) Full Test 1
Join Our Telegram Channel https://t.me/UPSCMaterials For Instant Updates

Answer: (b) compulsory contribution collected by the


Explanation: government from the public at large and
Chauth was a regular tax or tribute is to be used for a public purpose. On the
imposed, from early 18th century, by the other hand, a fee is imposed by the
Maratha Empire in India. It was an government for a specific facility or
annual tax nominally levied at 25% on service being provided or
revenue or produce, hence the name. It rendered. Cesses may be levied by the
was levied on the lands which were union or state governments. Cesses are
under nominal Mughal rule. named after the identified purpose; the
The sardeshmukhi was an additional 9- purpose itself must be certain and for
10 per cent of the land revenue paid to public good. Hence, only statement (c)
the head revenue collector in the is incorrect.
Deccan. Source:
Khanqahs or Hospice was a house of https://www.epw.in/engage/article/ces
rest for travellers, especially one kept by s-surcharge-distinction-significant-
a religious order. The Sufi masters held taxpayer
their assemblies in their khanqahs or
hospices. Devotees of all descriptions 56. Consider the following statements
including members of the royalty and with regard to Child Adoption
nobility, and ordinary people flocked to Regulatory Authority (CARA):
these khanqahs. They discussed spiritual (1) It functions as the nodal body
matters, sought the blessings of the for adoption of Indian
saints in solving their worldly problems, children and is mandated to
or simply attended the music and dance monitor and regulate in-
sessions. Hence, (b) is the correct country and inter-country
answer adoptions.
Source - NCERT: Class VII: Our Pasts- (2) Central Adoption Resource
II: Page No: 150 Authority (CARA) is a
statutory body of the Ministry
55. Which of the following statements of Women & Child
regarding Cess is incorrect? Development.
(a) It is a tax on tax. Which of the statements given
(b) It is levied for specific above is/are incorrect?
purposes. (a) 1 only
(c) It is levied only by the Union (b) 2 only
government. (c) Both 1 and 2
(d) Article 270 of the Constitution (d) Neither 1 nor 2
describes a cess.
Answer: (d)
Answer: (c) Explanation:
Explanation: Statement (1) is correct.
A cess is imposed by the government as Central Adoption Resource Authority
a tax on tax. A cess may be in the nature (CARA) functions as the nodal body for
of a tax or a fee but it is imposed for a adoption of Indian children and is
specific purpose, as identified in the mandated to monitor and regulate in-
charging legislation. Article 270 of the country and inter-country adoptions.
Constitution describes a cess. A tax is a CARA is designated as the Central

29
Prelim IAS Test Series (2019) – GS Test 14 (17.03.2019)
GS (Paper-I) Full Test 1
Join Our Telegram Channel https://t.me/UPSCMaterials For Instant Updates

Authority to deal with inter-country known as the "Coffee Club"; the L.69
adoptions in accordance with the Group of Developing Countries; the Arab
provisions of the Hague Convention on League and the Caribbean Community
Inter-country Adoption, 1993, ratified by (CARICOM).
the Government of India in 2003. Source: Vajiram and Ravi current
Statement (2) is correct. affairs magazine September
It is a statutory body of the Ministry of
Women & Child Development, 58. Consider the following statements
Government of India. with reference to Doppler radar
Source: Vajiram and Ravi current which was recently in news:
affairs magazine September (1) A Doppler radar is a
http://cara.nic.in/about/about_cara.ht specialized radar that uses
ml the Doppler Effect to produce
velocity data about objects at
57. The Intergovernmental a distance.
Negotiations framework (IGN) was (2) No Doppler radar has been
in news recently. It is related to installed in India yet.
which of the following? (3) It is used for weather
(a) It is a framework created forecasting only.
under the Paris climate deal Which of the statements given
to facilitate negotiations above is/are correct?
between developed and (a) 1 only
developing countries (b) 1 and 2 only
regarding emission reduction. (c) 2 and 3 only
(b) It is a group of nation-states (d) 1, 2 and 3
working within the United
Nations to further reform of Answer: (a)
the United Nations Security Explanation:
Council (UNSC). Statement (1) is correct.
(c) It is a tool available under the A Doppler radar is a specialized radar
World Trade Organisation that uses the Doppler Effect to produce
(WTO) to resolve trade velocity data about objects at a distance.
disputes among nations. It does this by bouncing a microwave
(d) None of the above signal off a desired target and analysing
how the object's motion has altered the
Answer: (b) frequency of the returned signal.
Explanation: Statement (2) is incorrect.
Option (b) is correct. The first Doppler radar was installed in
The Intergovernmental Negotiations Chennai in 2002 and currently there are
framework or IGN is a group of nation- 27 Doppler radars in the country.
states working within the United Nations Statement (3) is incorrect.
to further the reform of the United Doppler radars are used in weather
Nations Security Council (UNSC). It is forecasting, aviation, sounding satellites,
composed of several different meteorology etc.
international organizations, including the Source:
African Union; the G4 nations; the
Uniting for Consensus Group (UfC), also

30
Prelim IAS Test Series (2019) – GS Test 14 (17.03.2019)
GS (Paper-I) Full Test 1
Join Our Telegram Channel https://t.me/UPSCMaterials For Instant Updates

https://www.livemint.com/Science/VIlU Maharashtra, Karnataka and Andhra


Xxg4fhF9JaYceFsbYI/IMD-to-add-30- Pradesh.
Doppler-radars-in-country.html Statement (3) is correct.
https://www.sciencedaily.com/terms/d It has also been identified as one of the
oppler_radar.htm species for the recovery programme
under the Integrated Development of
59. Consider the following statements Wildlife Habitats of the Ministry of
with respect to the Great Indian Environment and Forests.
Bustard: Source:
(1) It is listed as Critically https://www.thehindu.com/sci-
Endangered in the IUCN Red tech/energy-and-environment/the-
List. indian-bustard-on-its-last-
(2) Currently, its population is legs/article25758411.ece
found only in Rajasthan and https://www.wwfindia.org/about_wwf/
Gujarat. priority_species/threatened_species/g
(3) It has been identified as one reat_indian_bustard/
of the species for the
recovery programme under 60. The seals of the Harappan period
the Integrated Development are made up of
of Wildlife Habitats of the (1) Steatite
Ministry of Environment and (2) Copper
Forests. (3) Gold
Which of the statements given (4) Ivory
above are correct? Select the correct answer using the
(a) 1 and 2 only code given below:
(b) 1 and 3 only (a) 1 and 2 only
(c) 2 and 3 only (b) 2 and 3 only
(d) 1, 2 and 3 (c) 1, 2 and 4 only
(d) 1, 2, 3 and 4
Answer: (b)
Explanation: Answer: (d)
Statement (1) is correct. Explanation:
The Great Indian Bustard is listed in Archaeologists have discovered
Schedule I of the Indian Wildlife thousands of seals belonging to
(Protection) Act, 1972, in the CMS Harrapan civilization, usually made of
Convention and in Appendix I of CITES, steatite, and occasionally of agate,
as Critically Endangered on the IUCN chert, copper, faience and terracotta,
Red List and the National Wildlife Action with beautiful figures of animals, such as
Plan (2002-2016). unicorn bull, rhinoceros, tiger, elephant,
Statement (2) is incorrect. bison, goat, buffalo, etc. The purpose of
Historically, the Great Indian Bustard was producing seals was mainly commercial.
distributed throughout Western India, It appears that the seals were also used
spanning 11 states, as well as parts of as amulets, carried on the persons of
Pakistan. Today, its population is their owners, perhaps as modern-day
confined mostly to Rajasthan and Gujarat identity cards. The standard Harappan
but small population also occur in seal was a square plaque 2×2 square
inches, usually made from the soft river

31
Prelim IAS Test Series (2019) – GS Test 14 (17.03.2019)
GS (Paper-I) Full Test 1
Join Our Telegram Channel https://t.me/UPSCMaterials For Instant Updates

stone, steatite. Some seals have also


been found in gold and ivory. Hence 62. Consider the following statements:
option (d) is the correct answer. (1) The philosophy of Shuddh
Source: An introduction to Indian ART Advaita was propounded by
Class XI Page: 11 Shankaracharya.
(2) Vishishtadvaita is a non-
61. Consider the following statements dualistic school of Vedanta
with reference to the philosophy propounded by
Hoysaleshvara temple at Halebid in Ramanuja.
Karnataka: Which of the statements given
(1) The temple was constructed above is/are correct?
during Hoysala rule and is (a) 1 only
dedicated to Lord Murugan. (b) 2 only
(2) It is one of the best example (c) Both 1 and 2
of Dravida style of (d) Neither 1 nor 2
architecture.
Which of the statements given Answer: (b)
above is/are correct? Explanation:
(a) 1 only Statement (1) is incorrect.
(b) 2 only Shankara, one of the most influential
(c) Both 1 and 2 philosophers of India, was born in Kerala
(d) Neither 1 nor 2 in the eighth century. He was an
advocate of Advaita or the doctrine of the
Answer: (d) oneness of the individual soul and the
Explanation: Supreme God which is the Ultimate
Statement (1) is incorrect. Reality. He taught that Brahman, the only
Hoysaleswara temple, is a 12th-century or Ultimate Reality, was formless and
Hindu temple dedicated to Lord Shiva. It without any attributes. He considered the
is the largest monument in Halebid, world around us to be an illusion or
Karnataka. The temple was built on the maya, and preached renunciation of the
banks of a large man-made lake, and world and adoption of the path of
sponsored by King Vishnuvardhana of knowledge to understand the true nature
the Hoysala Empire. Its construction of Brahman and attain salvation.
started around 1121 CE and was Vallabhcharya promoted the philosophy
complete in 1160 CE. The temple was of Pushtimarga. He was the founder of
built in dark schist stone. Hoysala Shuddhadvaita (purely non-dual).
temples are easily distinguishable from Statement (2) is correct.
other medieval temples by their highly Vishishtadvaita is a non-dualistic school
original star-like ground-plans and a of Vedanta philosophy propounded by
profusion of decorative carvings Ramanuja. He was born in Tamil Nadu in
Statement (2) is incorrect. the eleventh century, was deeply
The style of Hoysala temples is called influenced by the Alvars. According to
hybrid or Vesara style as their unique him the best means of attaining salvation
style seems neither completely dravida was through intense devotion to Vishnu.
nor nagara, but somewhere in between. Vishnu in His grace helps the devotee to
Source: An introduction to Indian ART attain the bliss of union with Him. He
Class XI Page: 89 propounded the doctrine of

32
Prelim IAS Test Series (2019) – GS Test 14 (17.03.2019)
GS (Paper-I) Full Test 1
Join Our Telegram Channel https://t.me/UPSCMaterials For Instant Updates

Vishishtadvaita or qualified oneness in The Sixth Guru Hargovind founded a


that the soul even when united with the palace opposite to Harmandir Sahib
Supreme God remained distinct. known as ‘Akal takht’. The Tenth Guru,
Ramanuja's doctrine greatly inspired the Govind Singh introduced ‘Panchkakar’ of
new strand of bhakti which developed in Sikhism - Kesh (long hair) Kanga (comb)
north India subsequently. Kripan (Sword), Kachha (Underwear)
Source: NCERT: Class VII: Our Pasts- and Kara (Iron bangk). He also formed
II: Page No: 107 the Khalsa for the Sikh brotherhood in
1699. Hence statements (2) and (3) are
63. Consider the following statements: correct.
(1) The fifth Sikh Guru Arjun Dev Source: NCERT: Class VII: Our Pasts-
introduced Gurumukhi Script II: Page No: 117-118
and also compiled Guru
Nanak’s biography Janam 64. Consider the following statements
Sakis. with reference to mass movements
(2) The Sixth Guru Hargovind in the geographical context:
founded a palace opposite to (1) These movements transfer
Harmandir Sahib known as the mass of rock debris down
‘Akal takht’. the slopes under the direct
(3) The tenth Guru, Govind influence of gravity.
Singh introduced ‘Panch (2) Weathering is a pre-requisite
Kakar’ and also formed the for mass movement.
Khalsa. (3) Mass movements are aided
Which of the statements given by geomorphic agent like
above is/are correct? running water, glaciers, wind,
(a) 1 and 2 only waves etc.
(b) 2 and 3 only Which of the statements given
(c) 1 and 3 only above is/are correct?
(d) 1, 2 and 3 (a) 1 only
(b) 1 and 2 only
Answer: (b) (c) 2 and 3 only
Explanation: (d) 1, 2 and 3
Guru Angad introduced Gurumukhi
Script. He also compiled Guru Nanak’s Answer: (a)
biography Janam Sakis. The three Explanation:
successors of Guru Angad also wrote Statement (1) is correct.
under the name of "Nanak" and all of Mass movements transfer the mass of
their compositions were compiled by rock debris down the slopes under the
Guru Arjan in 1604. To this compilation direct influence of gravity.
were added the writings of other figures Statement (2) is incorrect.
like Shaikh Farid, Sant Kabir, Bhagat Gravity exerts its force on all matter, both
Namdev and Guru Tegh Bahadur. In bedrock and the products of weathering.
1706 this compilation was authenticated So, weathering is not a pre-requisite for
by his son and successor. Guru Gobind mass movement though it aids mass
Singh. It is now known as Guru Granth movements. Mass movements are very
Sahib, the holy scripture of the Sikhs. active over weathered slopes rather than
Hence, statement (1) is incorrect. over unweathered materials.

33
Prelim IAS Test Series (2019) – GS Test 14 (17.03.2019)
GS (Paper-I) Full Test 1
Join Our Telegram Channel https://t.me/UPSCMaterials For Instant Updates

Statement (3) is incorrect. is the most important layer for all


Mass movements are aided by gravity biological activity.
and no geomorphic agent like running Statement (3) is correct.
water, glaciers, wind, waves and currents The Mesosphere lies above the
participate in the process of mass Stratosphere, which extends up to a
movements. That means mass height of 80 km. In this layer,
movements do not come under erosion temperature starts decreasing with the
though there is a shift (aided by gravity) increase in altitude and reaches up to
of materials from one place to another. minus 100°C at the height of 80 km.
Source: Fundamentals Of Physical Source: Fundamentals of Physical
Geography, page: 52 Geography, page: 77

65. Consider the following statements 66. With reference to the natural
with reference to the structure of vegetation of India, consider the
atmosphere: following statements:
(1) Troposphere is the (1) Deodar, a highly valued
lowermost layer of the endemic species grows
atmosphere and contains mainly in the western part of
ozone layer. the Himalayan range.
(2) Stratosphere is the most (2) Shola forests are found
important layer for all mainly in the north eastern
biological activity. states along the foothills of
(3) In Mesosphere, temperature Himalayas.
starts decreasing with (3) Wild Date Palm and Neem
increase in altitude. are found in the moist
Which of the statements given deciduous forest.
above is/are incorrect? Which of the statements given
(a) 1 only above is/are correct?
(b) 1 and 2 only (a) 1 only
(c) 2 and 3 only (b) 1 and 3 only
(d) 1, 2 and 3 (c) 2 and 3 only
(d) 1, 2 and 3
Answer: (b)
Explanation: Answer: (a)
Statement (1) is incorrect. Explanation:
The Stratosphere is found above the Statement (1) is correct.
Tropopause and extends up to a height Deodar, a highly valued endemic species
of 50 km. One important feature of the grows mainly in the western part of the
stratosphere is that it contains the ozone Himalayan range. Deodar is a durable
layer. This layer absorbs ultra-violet wood mainly used in construction activity.
radiation and shields life on the Earth The Moist deciduous forests are more
from intense, harmful form of energy. pronounced in the regions which record
Statement (2) is incorrect. rainfall between 100-200 cm. These
The Troposphere is the lowermost layer forests are found in the north eastern
of the atmosphere. All changes in climate states along the foothills of Himalayas,
and weather take place in this layer. This eastern slopes of the Western Ghats and
Odisha. Teak, sal, shisham, hurra,

34
Prelim IAS Test Series (2019) – GS Test 14 (17.03.2019)
GS (Paper-I) Full Test 1
Join Our Telegram Channel https://t.me/UPSCMaterials For Instant Updates

mahua, amla, semul, kusum, and Source: Fundamentals of Physical


sandalwood etc. are the main species of Geography, page: 59-67
these forests.
Statement (2) is incorrect. 68. Consider the following statements
Shola forests are the patches of stunted regarding the effects of national
tropical montane forest found in valleys emergency under Article 352 of the
within rolling grassland in the montane Indian Constitution:
regions of South India. The temperate (1) During a national emergency,
forests are called Sholas in the Nilgiris, the state governments are
Anaimalai and Palami Hills. brought under the complete
Statement (3) is incorrect. control of the Centre.
Tropical thorn forests occur in the (2) The Parliament becomes
areas which receive rainfall less than 50 empowered to make laws on
cm. These consist of a variety of grasses any subject in the State List
and shrubs. In these forests, plants and the legislative power of a
remain leafless for most part of the year State Legislature is
and give an expression of scrub suspended, until the time
vegetation. Important species found are national emergency is in
babool, ber, and wild date palm, khair, force.
neem, khejri, palas, etc. Which of the statements given
Source: INDIA: PHYSICAL above is/are correct?
ENVIRONMENT Page 59-60. (a) 1 only
(b) 2 only
67. Match the Landforms with the (c) Both 1 and 2
Geomorphic agent due to which (d) Neither 1 nor 2
they are formed –
Landforms Geomorphic Answer: (a)
Agent Explanation:
A. Mushroom 1. River Statement (1) is correct.
rocks During a national emergency, the
B. Cirque 2. Ground water executive power of the Centre extends to
C. Stalagmites 3. Glacier directing any State regarding the manner
D. Monadnocks 4. Wind in which its executive power is to be
Codes exercised. In normal times, the Centre
A B C D can give executive directions to a State
(a) 4 3 2 1 only on certain specified matters.
(b) 1 2 3 4 However, during a national emergency,
(c) 1 4 2 3 the Centre becomes entitled to give
(d) 2 3 1 4 executive directions to a State on ‘any’
matter. Thus, the State governments are
Answer: (a) brought under the complete control of the
Explanation: Centre, though they are not suspended.
Landforms Geomorphic agent Statement (2) is incorrect.
1. Mushroom rocks A) Wind During a national emergency, the
2. Cirque: B) Glacier Parliament becomes empowered to
3. Stalagmites C) Ground water make laws on any subject mentioned in
4. Monadnocks: D) River the State List. However, the legislative

35
Prelim IAS Test Series (2019) – GS Test 14 (17.03.2019)
GS (Paper-I) Full Test 1
Join Our Telegram Channel https://t.me/UPSCMaterials For Instant Updates

power of a State Legislature is not 70. Consider the following statements


suspended, but it becomes subject to the with respect to the Reserve Bank
overriding power of the Parliament. of India’s Monetary Policy
Source: Indian Polity- M Laxmikanth, Committee (MPC):
4th Edition ch-16; Emergency (1) It is a six-member panel
provisions; page- 16.3 having four members from
the RBI, and two
69. Which of the following come under independent members
the legislative powers of the selected by the Union
President of India? Government.
(1) He can make rules for more (2) The MPC will meet four times
convenient transaction of a year and take its monetary
business of the Union policy decisions based on the
government, and for multiple indicator approach.
allocation of the said Which of the statements given
business among the above is/are correct?
ministers. (a) 1 only
(2) He can summon or prorogue (b) 2 only
the Parliament and dissolve (c) Both 1 and 2
the Lok Sabha. (d) Neither 1 nor 2
(3) He can declare any area as
scheduled area and has Answer: (d)
powers with respect to the Explanation:
administration of scheduled Statement (1) is incorrect.
areas and tribal areas. The Monetary Policy Committee (MPC)
Select the correct answer using the is a Committee of the Central Bank in
code given below: India (Reserve Bank of India), headed by
(a) 1 and 2 only its Governor, which is entrusted with the
(b) 2 only task of fixing the benchmark policy
(c) 1 and 3 only interest rate (repo rate) to contain
(d) None of the above inflation within the specified target level.
The new MPC is a six-member panel that
Answer: (b) is expected to bring “value and
Explanation: transparency” to rate-setting decisions. It
Statement (1) and Statement (3) are features three members from the RBI —
incorrect as these come under the the Governor, a Deputy Governor and
Executive powers of president. another official — and three independent
Statement (2) is correct. members to be selected by the Union
Summoning or proroguing the Parliament Government. A search committee
and dissolving the Lok Sabha comes recommends three external members,
under the legislative power of the experts in the field of economics, banking
President. or finance, for the Government
Source: Indian Polity- M Laxmikanth, appointees.
4th Edition ch-17; President; page- Statement (2) is incorrect.
17.7 The MPC meets four times a year to
decide on monetary policy by a majority
vote. And if there’s a tie, the RBI

36
Prelim IAS Test Series (2019) – GS Test 14 (17.03.2019)
GS (Paper-I) Full Test 1
Join Our Telegram Channel https://t.me/UPSCMaterials For Instant Updates

Governor gets the deciding vote. Until both the Lok Sabha and the
recently, India’s central bank used to Rajya Sabha.
take its monetary policy decisions based (2) It should cover more than one
on the multiple indicator approach. Its matter of urgent public
rate decisions were expected to take into importance and the
account inflation, growth, employment, discussion on the motion
banking stability and the need for a should last for not less than
stable exchange rate. Thus, the RBI (with two hours and thirty minutes.
the Governor as the focal point) would be (3) It involves an element of
subject to hectic lobbying ahead of each censure against the
policy review and trenchant criticism after government.
it. To resolve this, the RBI set up an Which of the statements given
Expert Committee under Urijit Patel to above is/are correct?
revise the monetary policy framework. It (a) 1 only
suggested that the RBI abandon the (b) 2 and 3 only
‘multiple indicator’ approach and make (c) 3 only
inflation targeting the primary (d) 1, 2 and 3
objective of its monetary policy. It also
mooted having an MPC so that these Answer: (c)
decisions could be made through Explanation:
majority vote. Having both Government Adjournment Motion is introduced in the
and the RBI members on the MPC for Parliament to draw attention of the
accountability, the Government would House to a definite matter of urgent
have to keep its deficit under check and public importance, and needs the support
the RBI would owe an explanation for of 50 members to be admitted. As it
runaway inflation. interrupts the normal business of the
Source: House, it is regarded as an extraordinary
https://www.thehindubusinessline.co device. It involves an element of censure
m/opinion/columns/slate/all-you- against the government and hence Rajya
wanted-to-know-about-monetary- Sabha is not permitted to make use of
policy-committee/article8807786.ece this device. Hence, statement (1) is
http://www.arthapedia.in/index.php?tit incorrect and statement (3) is correct.
le=Monetary_Policy_Committee_(MPC The discussion on an adjournment
) motion should last for not less than two
http://pib.nic.in/newsite/PrintRelease.a hours and thirty minutes. However, the
spx?relid=151264 right to move a motion for an
adjournment of the business of the
71. In the Parliament of India, the House is subject to the following
purpose of an adjournment motion restrictions:
is to allow a discussion on a 1. It should raise a matter which is
definite matter of urgent public definite, factual, urgent and of public
importance. With reference to the importance;
Adjournment Motion, consider the 2. It should not cover more than one
following statements: matter; (Hence, Statement 2 is
(1) It needs the support of 50 incorrect)
members to be admitted in 3. It should be restricted to a specific
matter of recent occurrence and

37
Prelim IAS Test Series (2019) – GS Test 14 (17.03.2019)
GS (Paper-I) Full Test 1
Join Our Telegram Channel https://t.me/UPSCMaterials For Instant Updates

should not be framed in general from the Lok Sabha and 7 from the Rajya
terms; Sabha).
4. It should not raise a question of Statement (2) is correct.
privilege; The Public Accounts Committee
5. It should not revive discussion on a examines the money spent on any
matter that has been discussed in the service during a financial year in excess
same session of the amount granted by the Lok Sabha
6. It should not deal with any matter that for that purpose.
is under adjudication by court; and Statement (3) is correct.
7. It should not raise any question that The effectiveness of the role of Public
can be raised on a distinct motion. Accounts Committee is limited by the
Source: Indian Polity- M Laxmikanth, following:
4th Edition ch-22; Parliament; page- (a) It is not concerned with the questions
22.15 of policy in a broader sense.
(b) It conducts a post-mortem
72. Consider the following statements examination of accounts (showing
with reference to the Public the expenditure already incurred).
Accounts Committee: (c) It cannot intervene in the matters of
(1) This Committee was set up day-to-day administration.
under the provisions of the (d) Its recommendations are advisory
Government of India Act of and not binding on the ministries.
1935. (e) It is not vested with the power of
(2) It examines the money spent disallowance of expenditures by the
on any service during a departments.
financial year in excess of the (f) It is not an executive body and
amount granted by the Lok hence, cannot issue an order. Only
Sabha for that purpose. the Parliament can take a final
(3) It is not vested with the decision on its findings.
power of disallowance of Source: Indian Polity- M Laxmikanth,
expenditures by the 4th Edition ch-22; Parliament; page-
departments. 22.30
Which of the statements given
above is/are correct? 73. Which of the following comes
(a) 1 and 3 only under the voluntary provisions of
(b) 2 only the 73rd Constitutional Amendment
(c) 2 and 3 only Act, 1992?
(d) 1, 2 and 3 (1) The organisation of Gram
Sabha in a village or group of
Answer: (c) villages.
Explanation: (2) Indirect elections to the post
Statement (1) is incorrect. of chairperson of Panchayats
The Public Accounts Committee was set at the intermediate and
up first in 1921 under the provisions of district levels.
the Government of India Act of 1919 (3) Reservation of one-third
and has since been in existence. At seats (both members and
present, it consists of 22 members (15 chairpersons) for women in

38
Prelim IAS Test Series (2019) – GS Test 14 (17.03.2019)
GS (Paper-I) Full Test 1
Join Our Telegram Channel https://t.me/UPSCMaterials For Instant Updates

panchayats at all the three 74. With reference to the Quit India
levels. Movement, consider the following
Select the correct answer using the statements:
code given below: (1) Usha Mehta organized the
(a) 2 only Secret Congress Radio.
(b) 2 and 3 only (2) Chittu Pande formed parallel
(c) 1 and 3 only government in Ballia.
(d) None of the above (3) Khan Abdul Gaffar Khan
formed a clan of non-violent
Answer: (d) revolutionaries, the Khudai
Explanation: Khidmatgars.
All of the given options come under the Which of the statements given
compulsory (obligatory or mandatory) above is/are correct?
provisions of the 73rd Constitutional (a) 1 only
Amendment Act (1992). (b) 1 and 2 only
Voluntary Provisions of the 73rd (c) 3 only
Constitutional Amendment Act (1992) (d) 1, 2 and 3
includes:
1. Giving representation to members of Answer: (b)
the Parliament (both the Houses) Explanation:
and the State Legislature (both the Statement (1) is correct.
Houses) in the Panchayats at Usha Mehta actively supported the Quit
different levels falling within their India Movement, and was an important
constituencies. member of a small group which ran the
2. Providing reservation of seats (both Congress Radio.
members and chairpersons) for Statement (2) is correct.
backward classes in Panchayats at Chittu Pande, who called himself a
any level. Gandhian, formed a parallel government
3. Granting powers and authority to the and captured all the ten police stations in
Panchayats to enable them to Ballia, in east UP in August 1942.
function as institutions of self- Statement (3) is incorrect.
government. Khan Abdul Gaffar Khan formed a clan of
4. Devolution of powers and non-violent revolutionaries, the Khudai
responsibilities upon Panchayats to Khidmatgars (known as Red Shirts), who
prepare plans for economic played an active role in Civil
development and social justice; and disobedience movement.
to perform some or all of the Source: A brief History of Modern
functions listed in the Eleventh India Page 363&367
Schedule of the Constitution.
5. Granting financial powers to the 75. The reforms of Lord William
Panchayats, that is, authorizing them Bentinck included which of the
to levy, collect and appropriate following?
taxes, duties, tolls and fees. (1) Introduction of English
Source: Indian Polity- M Laxmikanth, language as an official
4th Edition ch-34; Panchayati Raj; language.
page- 34.10 (2) Abolition of Provincial Courts.
(3) Abolition of Sati.

39
Prelim IAS Test Series (2019) – GS Test 14 (17.03.2019)
GS (Paper-I) Full Test 1
Join Our Telegram Channel https://t.me/UPSCMaterials For Instant Updates

(4) Suppression of thuggee. (3) India became a member of


Select the correct answer using the IEA in 2017.
code given below: Which of the statements given
(a) 1 and 3 only above are correct?
(b) 1, 2 and 3 only (a) 1 and 2 only
(c) 2, 3 and 4 only (b) 2 and 3 only
(d) 1, 2, 3 and 4 (c) 1 and 3 only
(d) 1, 2 and 3
Answer: (d)
Explanation: Answer: (a)
Reforms of Lord William Bentinck (1828- Explanation:
1835) included Statement (1) is correct.
(i) Abolition of sati and other cruel rites The International Energy Agency is a
(1829). Paris-based autonomous
(ii) Suppression of thugi (1830). intergovernmental organization
(iii) Charter Act of 1833. established in the framework of the
(iv) Resolution of 1835, and educational Organisation for Economic Co-operation
reforms and introduction of English and Development in 1974 in the wake of
as the official language. the 1973 oil crisis.
(iv) Annexation of Mysore (1831), Coorg Statement (2) is correct.
(1834) and Central Cachar (1834). A candidate country must be a member
(v) Treaty of 'perpetual friendship with country of the OECD. However,
Ranjeet Singh. membership in the OECD does not
(vi) Abolition of the provincial courts of automatically result in membership in the
appeal and circuit set up by IEA.
Cornwallis, appointment of Statement (3) is incorrect.
commissioners of revenue and India is not a member of IEA but in
circuit. March 2017, after a series of intensive
Hence (d) is the correct answer. consultations with all the relevant
Source: A brief History of Modern ministries, India joined the IEA as an
India Page 370. Association country.
Source:
76. With reference to International https://www.iea.org/about/history/
Energy Agency (IEA) recently in https://www.iea.org/countries/India/
news, consider the following https://www.financialexpress.com/mar
statements: ket/commodities/uae-wants-to-store-
(1) It is a Paris-based crude-oil-in-india-keen-to-invest-in-
autonomous refining-petrochemical-
intergovernmental projects/1484108/
organization established in
the framework of the 77. The term 'anthanar' in Sangam age
Organisation for Economic was used to denote
Co-operation and (a) Priest
Development (OECD). (b) Military head
(2) Only an OECD member (c) Ministers
country can become the (d) Envoy
member of IEA.

40
Prelim IAS Test Series (2019) – GS Test 14 (17.03.2019)
GS (Paper-I) Full Test 1
Join Our Telegram Channel https://t.me/UPSCMaterials For Instant Updates

Answer: (a) was Chandragupta I (320 – 330 A.D.)


Explanation: The Sangam Age and he was the first to be called
constitutes an important chapter in the Maharajadhiraja (the great king of kings).
history of South India. According to Tamil Statement (2) is correct.
legends, there existed three Sangams The Chinese traveller Fahien, who visited
(Academy of Tamil poets) in ancient India during the reign of Chandragupta II,
Tamil Nadu popularly called has left a valuable account of the social,
Muchchangam. These Sangams economic and religious conditions of the
flourished under the Royal patronage of Gupta Empire.
the Pandyas. Statement (3) is incorrect.
Sangam Polity The decline of the Gupta empire was
Terms Used for followed by a period of political disorder
Amaichar - Ministers and disunity in North India. It was only in
Anthanar- Priests the beginning of the seventh century A.D.
Military commanders - Senapathi that Harshvardhana succeeded in
Thuthar- Envoys establishing a larger kingdom in north
Orrar- spies India. Hiuen Tsang gives a detailed
Source: Lesson 8 Sangam picture about administration of Harsha.
Age, Standard XI - History Tamil Source: Tamil Nadu Standard XI
Nadu (page no. 89) Lesson 9, Gupta Empire (page no. 96,
113, 116)
78. Consider the following statements
about Gupta empire: 79. Consider the following statements
(1) The founder of the Gupta with reference to the Start Up India
dynasty was Sri Gupta. Scheme:
(2) The Chinese traveller Fahien, (1) According to the scheme, a
visited India during the reign startup is an entity that is
of Chandragupta II headquartered in India which
(3) Hiuen Tsang gives a detailed was opened less than five
picture about the years ago and has an annual
administration of turnover less than 25 crore.
Kumaragupta. (2) Rural India’s version of Start-
Which of statements given above up India was named the
is/are correct? Deen Dayal Upadhyay
(a) 1 and 2 only Swaniyojan Yojana.
(b) 2 only Which of the statements given
(c) 1 and 3 only above is/are correct?
(d) 1, 2 and 3 (a) 1 only
(b) 2 only
Answer: (a) (c) Both 1 and 2
Explanation: (d) Neither 1 nor 2
Statement (1) is correct.
The founder of the Gupta dynasty was Answer: (c)
Sri Gupta. He was succeeded by Explanation:
Ghatotkacha. These two were called The Start Up India Scheme aims at
Maharajas. Not much information is fostering entrepreneurship and promoting
available about their rule. The next ruler innovation by creating an ecosystem that

41
Prelim IAS Test Series (2019) – GS Test 14 (17.03.2019)
GS (Paper-I) Full Test 1
Join Our Telegram Channel https://t.me/UPSCMaterials For Instant Updates

is conducive for growth of Start-ups. Upgradation of Government


According to the scheme, a startup is an Medical College Institutions.
entity that is headquartered in India Which of the statements given
which was opened less than five years above is/are correct?
ago and has an annual turnover less (a) 1 only
than 25 crore. It provides Simple (b) 2 only
Compliance Regime for startups based (c) Both 1 and 2
on Self-certification. Single window (d) Neither 1 nor 2
clearance based on mobile App. Startup
India Hub to handhold startups during Answer: (b)
various phases of their development. Explanation:
Legal support and fast-track patent Statement (1) is incorrect and is related
examination by reducing 80% of the to Janani Suraksha Yojana. The main
patent cost. Faster exit for startups objective of the JSY is to reduce
through modified new bankruptcy code maternal and infant mortality by
ensuring 90 days exit window. Credit promoting institutional delivery among
Guarantee Fund for startups through pregnant women by providing conditional
Small Industries Development Bank of cash assistance of Rs. 1400.
India (SIDBI). Rural India’s version of Statement (2) is correct.
Startup India was named the Deen Dayal The Pradhan Mantri Swasthya Suraksha
Upadhyay Swaniyojan Yojana. Hence, Yojana (PMSSY) was announced in
both statements are correct. 2003 with objectives of correcting
Source: regional imbalances in the availability of
https://www.startupindia.gov.in/conte affordable/ reliable tertiary healthcare
nt/sih/en/startup-scheme.html services and also to augment facilities for
https://economictimes.indiatimes.com quality medical education in the country.
/small-biz/startups/startup-india- The main two components of PMSSY is
initiative-to-get-a-rural-avatar-as-deen- setting up of AIIMS like Institutions and
dayal-upadhyay-swaniyojan- Upgradation of Government Medical
yojana/articleshow/51576609.cms College Institutions.
Source: http://pmssy-
80. Consider the following statements mohfw.nic.in/listofcolleges.aspx
with reference to the Pradhan
Mantri Swasthya Suraksha 81. Consider the following Modes of
Yojana (PMSSY): Trade in Services under General
(1) The main objective of the Agreement on Trade in Services
PMSSY is to reduce maternal (GATS):
and infant mortality by Modes Services
promoting institutional 1. Mode 1 Cross-border
delivery among pregnant supply
women by providing 2. Mode 2 Consumption
conditional cash assistance abroad
of Rs. 1400. 3. Mode 3 Presence of a
(2) The two main components of natural person
PMSSY are setting up of
AIIMS like Institutions and Which of the pairs given above
is/are correctly matched?

42
Prelim IAS Test Series (2019) – GS Test 14 (17.03.2019)
GS (Paper-I) Full Test 1
Join Our Telegram Channel https://t.me/UPSCMaterials For Instant Updates

(a) 1 only (b) Movement in opposition to


(b) 1 and 2 only the possible expansion of
(c) 2 and 3 only permanent seats in the
(d) 1, 2 and 3 UNSC.
(c) The United Kingdom
Answer: (b) European Union
Explanation: membership referendum
General Agreement on Trade in Services (d) World's largest global airline
(GATS) is the first agreement on alliance
multilateral trade in services. It divides
tradable services into four modes on the Answer: (a)
basis of requirement of commercial or Explanation:
physical presence of service provider A unicorn, in the world of business, is a
from the exporting country company, usually a start-up that does not
(i) Mode 1 or Cross-Border Supply - have an established performance record,
These are services which can be with a stock market valuation or
exported without need of commercial or estimated valuation of more than $1
physical presence of service provider in billion. The term was coined in 2013 by
the other country, e g. IT, BPO Services, venture capitalist Aileen Lee choosing
Air Services etc. the mythical animal to represent the
(ii) Mode 2 or Consumption Abroad - It is statistical rarity of such successful
similar to Mode 1 as no presence of ventures. Recently, India saw eight start-
service provider needed in other country, ups enter the unicorn club. Hence, (a) is
but here the service consumer of the the correct answer
importing country has to consume it in Source: https://www.business-
the exporting country, e g. Tourism standard.com/article/companies/8-
(iii) Mode 3 or Commercial Presence - start-up-firms-turn-into-unicorns-in-
To export these services, at least 2018-more-likely-in-2019-
commercial presence of service provider 118122900668_1.html
is needed through offices etc. eg https://www.businessinsider.in/online-
Banking, Telecommunications. tutoring-startup-byjus-is-the-latest-
(iv) Mode 4 or Presence of Natural indian-unicorn-going-
Persons - These services cannot be global/articleshow/67569325.cms
exported unless the service providers
physically cross the border to provide 83. With reference to ‘Capital Gains
services, e g. Skilled professionals like Tax’, consider the following
engineers, doctors etc providing services statements:
to other countries. Hence, (b) is the (1) It refers to the tax on profits
correct answer. earned by investors on the
Source: Vajiram: Indian Economy sale of capital assets.
2019: Chapter 9: International (2) Capital gains are not
Institutions: Page No 99. applicable when an asset is
inherited.
82. ‘Unicorn club’ sometimes seen in (3) In India, the long-term capital
the news is related to gains on sale of listed
(a) Start-ups valued at $1 billion securities exceeding Rs. 1
and above lakh are taxed at 10% and

43
Prelim IAS Test Series (2019) – GS Test 14 (17.03.2019)
GS (Paper-I) Full Test 1
Join Our Telegram Channel https://t.me/UPSCMaterials For Instant Updates

short-term gains will be taxed 2. Strait of : Red Sea & Gulf of


at 15 percent as per the Hormuz Aden
Union Budget 2018. 3. Strait of : Java Sea &
Which of the statements given Sunda Indian Ocean
above are correct? 4. Strait of : Gulf of Persia &
(a) 1 and 2 only Bab-el- Gulf of Oman
(b) 1 and 3 only Mandab
(c) 2 and 3 only Which of the pairs given above are
(d) 1, 2 and 3 correctly matched?
(a) 1 and 3 only
Answer: (d) (b) 1 and 2 only
Explanation: (c) 3 and 4 only
Capital gains refer to the profits earned (d) None of the above
by investors on the sale of capital assets.
Such a profit is taxed under the head Answer: (a)
‘Income from Capital Gains. The transfer Explanation:
of capital asset must be made in the A strait is a naturally formed, narrow,
previous year. typically navigable waterway that
Short-Term Capital Asset: When the connects two larger bodies of water.
shares and securities are held by the
taxpayer for a period not more than 36
months preceding the date of its transfer,
it will be treated as a short-term capital
asset.
Long-Term Capital Asset: If the
taxpayer holds the shares and securities
for a period exceeding 36 months before
the transfer, it will be treated as a long-
term capital asset.
Capital gains are not applicable when an
asset is inherited because there is no
sale, only a transfer. In India, the long-
term capital gains on sale of listed
securities exceeding Rs. 1 lakh are taxed
at 10% and short-term gains will be taxed
at 15 percent as per the Union Budget
2018. Hence, all the statements are
correct.
Source:
https://www.bankbazaar.com/tax/capit
al-gains-tax.html

84. Consider the following pairs:


Strait Connects

1. Strait of : Andaman Sea &


Malacca South China Sea

44
Prelim IAS Test Series (2019) – GS Test 14 (17.03.2019)
GS (Paper-I) Full Test 1
Join Our Telegram Channel https://t.me/UPSCMaterials For Instant Updates

Strait Connects genocide, crimes against humanity and


1. Strait of Malacca: Andaman Sea & war crimes.
South China Sea Statement (2) is correct.
2. Strait of Hormuz: Gulf of Persia & The Court was established by the Rome
Gulf of Oman Statute. ICC came into force on 1 July
3. Strait of Sunda: Java Sea & Indian 2002 and is located in the Hague, The
Ocean Netherlands.
4. Strait of Bab-el-Mandab: Red Sea & Statement (3) is incorrect.
Gulf of Aden India is not a member of the International
Source: Criminal Court.
https://www.britannica.com/place/Stra Source: Vajiram and Ravi current
it-of-Malacca affairs magazine September
https://www.britannica.com/place/Stra
it-of-Hormuz 86. Consider the following statements
https://www.britannica.com/place/Sun with respect to the Ordinance
da-Strait making power of the President:
https://www.britannica.com/place/Bab (1) The power of the President
-El-Mandeb-Strait to legislate by Ordinance is a
parallel power of legislation.
85. Consider the following statements (2) Ordinance-making is a
with reference to the International discretionary power of the
Criminal Court (ICC): President.
(1) It is an independent judicial (3) An Ordinance can be
body with jurisdiction over retrospective, that is, it may
persons charged with come into force from a back
genocide, crimes against date.
humanity and war crimes. Which of the statements given
(2) The Court was established above is/are correct?
by the Rome Statute and (a) 1 and 2 only
came into force in 2002. (b) 1 and 3 only
(3) India is a founding member (c) 3 only
of the International Criminal (d) 1, 2 and 3
Court.
Which of the statements given Answer: (c)
above is/are correct? Explanation:
(a) 1 only Statement (1) is incorrect.
(b) 1 and 2 only President can promulgate an ordinance
(c) 2 and 3 only only when both the Houses of Parliament
(d) 1 and 3 only are not in session or when either of the
two Houses of Parliament is not in
Answer: (b) session. An ordinance can also be
Explanation: issued when only one House is in
Statement (1) is correct. session because a law can be passed by
The International Criminal Court (ICC) is both the Houses and not by one House
an independent judicial body with alone. An ordinance made when both the
jurisdiction over persons charged with Houses are in session is void. Thus, the
power of the President to legislate by

45
Prelim IAS Test Series (2019) – GS Test 14 (17.03.2019)
GS (Paper-I) Full Test 1
Join Our Telegram Channel https://t.me/UPSCMaterials For Instant Updates

ordinance is not a parallel power of Statement (1) is incorrect and


legislation. Statement (2) is correct.
Statement (2) is incorrect. The procedure for the amendment of the
President’s power of ordinance-making is Constitution is laid down in Article 368.
not a discretionary power, and he can An amendment of the Constitution can
promulgate or withdraw an ordinance be initiated only by the introduction of a
only on the advice of the council of Bill for the purpose in either Houses of
ministers headed by the prime minister. the Parliament and not in the State
Statement (3) is correct. Legislatures. The Bill can be introduced
An ordinance like any other legislation, either by a minister or by a private
can be retrospective, that is, it may come member and does not require prior
into force from a back date. It may modify permission of the President.
or repeal any act of Parliament or Statement (3) is incorrect.
another ordinance. It can alter or amend If the Bill seeks to amend the federal
a tax law also. provisions of the Constitution, it must
Source: Indian Polity- M Laxmikanth, also be ratified by the Legislatures of half
4th Edition ch-17; President; page- of the States by a simple majority, that
17.11 is, a majority of the members of the
House present and voting.
87. With reference to the amendment Source: Indian Polity- M Laxmikanth,
process of the Indian Constitution, 4th Edition ch-10; Amendment of the
consider the following statements: constitution; page- 10.1, 10.2
(1) An amendment to the
Constitution can be initiated 88. Which among the following is the
by the introduction of a Bill best description of ‘shalabhoga’
for the purpose in either according to Chola inscriptions?
Houses of the Parliament or (a) Land of non-Brahmana
the State Legislatures. peasant proprietors
(2) A Constitutional Amendment (b) Land for the maintenance of
Bill can be introduced by a a school
private member. (c) Land donated to Jaina
(3) If the Bill seeks to amend the institutions
federal provisions of the (d) Land gifted to temples
Constitution, it must also be
ratified by the Legislatures of Answer: (b)
half of the States by a special Explanation:
majority. Chola inscriptions mention several
Which of the statements given categories of land:
above is/are correct? Vellanvagai: land of non-Brahmana
(a) 1 and 2 only peasant proprietors
(b) 2 only Brahmadeya: land gifted to Brahmanas
(c) 1 and 3 only Shalabhoga: land for the maintenance of
(d) 2 and 3 only a school
Devadana, tirunamattukkani: land
Answer: (b) gifted to temples
Explanation: Pallichchhandam: land donated to Jaina
institutions

46
Prelim IAS Test Series (2019) – GS Test 14 (17.03.2019)
GS (Paper-I) Full Test 1
Join Our Telegram Channel https://t.me/UPSCMaterials For Instant Updates

Hence (b) is the correct answer. respect of which a guarantee has


Source: NCERT: Class VII: Our Pasts- been given by the Centre.
II: Page No: 26 Hence 1st statement is incorrect and
2nd statement is correct.
89. Consider the following statements Source: Indian Polity- M Laxmikanth,
with reference to borrowing by the 4th Edition ch-14; Centre-state
Indian States: relations; page- 14.11
(1) A State government can
borrow within India and 90. Consider the following statements
abroad upon the security of with reference to alluvial soils:
the Consolidated Fund of the (1) The alluvial soils vary in
State. nature from sandy loam to
(2) A State cannot raise any loan clay.
without the consent of the (2) They are generally rich in
Centre, if any part of the loan potash and phosphorous.
is still outstanding which was (3) Khadar represents a system
made to the State by the of older alluvium, deposited
Centre. away from the flood plains.
Which of the statements given Which of the statements given
above is/are correct? above is/are correct?
(a) 1 only (a) 1 only
(b) 2 only (b) 1 and 2 only
(c) Both 1 and 2 (c) 2 only
(d) Neither 1 nor 2 (d) 2 and 3 only

Answer: (b) Answer: (a)


Explanation: Explanation:
The Constitution makes the following Statement (1) is correct.
provisions with regard to the borrowing Alluvial soils cover about 40 per cent of
powers of the Centre and the states: the total area of the country. They are
• The Central government can borrow depositional soils, transported and
either within India or outside upon the deposited by rivers and streams. The
security of the Consolidated Fund of alluvial soils vary in nature from sandy
India or can give guarantees, but loam to clay.
both within the limits fixed by the Statement (2) is incorrect.
Parliament. However, a state Alluvial soils are generally rich in potash
government can borrow within India but poor in phosphorous.
and not abroad upon the security of Statement (3) is incorrect.
the Consolidated Fund of the State or In the Upper and Middle Ganga plain,
can give guarantees, but both within two different types of alluvial soils have
the limits fixed by the legislature of developed, viz. Khadar and Bhangar.
that state. Khadar is the new alluvium and is
• A State cannot raise any loan without deposited by floods annually, which
the consent of the Centre, if there is enriches the soil by depositing fine silts.
still outstanding any part of a loan Bhangar represents a system of older
made to the State by the Centre or in alluvium, deposited away from the flood
plains.

47
Prelim IAS Test Series (2019) – GS Test 14 (17.03.2019)
GS (Paper-I) Full Test 1
Join Our Telegram Channel https://t.me/UPSCMaterials For Instant Updates

Source: India: Physical Environment, Source: Vajiram and Ravi current


Page 70 affairs magazine November.

91. Consider the following statements 92. Consider the following statements
with regard to Swadesh Darshan with reference to the International
Scheme: Telecommunications Union (ITU):
(1) It is one of the flagship (1) ITU is the United Nations'
schemes of the Ministry of specialized agency for
Culture for development of Information and
thematic circuits in the Communication
country in a planned and Technologies – ICTs.
prioritised manner. (2) It is headquartered at
(2) It intends to promote cultural Geneva, Switzerland.
and heritage value of the (3) ITU (South Asia) Area Office
country by developing world- and Technology Innovation
class infrastructure in the Centre will be set up in New
circuit destinations identified Delhi.
under the scheme. Which of the statements given
Which of the statements given above are correct?
above is/are correct? (a) 1 and 2 only
(a) 1 only (b) 1 and 3 only
(b) 2 only (c) 2 and 3 only
(c) Both 1 and 2 (d) 1, 2 and 3
(d) Neither 1 nor 2
Answer: (d)
Answer: (b) Explanation:
Explanation: ITU is the United Nations' specialized
Statement (1) is incorrect. agency for information and
Swadesh Darshan Scheme is one of the communication technologies – ICTs. It is
flagship schemes of the Ministry of headquartered at Geneva, Switzerland. It
Tourism for development of thematic has 12 regional and area offices in the
circuits in the country in a planned and world. It allocates global radio spectrum
prioritised manner. and satellite orbits, develop the technical
Statement (2) is correct. standards that ensure networks and
It intends to promote cultural and technologies seamlessly interconnect,
heritage value of the country by and strive to improve access to ICTs to
developing world-class infrastructure in underserved communities worldwide.
the circuit destination. Fifteen thematic Recently, ITU has decided to set up the
circuits have been identified under this ITU South Asia Area Office and
scheme, for development namely: North- Technology Innovation Centre in New
East Circuit, Buddhist Circuit, Tirthankar Delhi. Hence all of the statements
Circuit, Himalayan Circuit, Coastal given above are correct.
Circuit, Krishna Circuit, Desert Circuit, Source: Vajiram and Ravi current
Tribal Circuit, Eco Circuit, Wildlife Circuit, affairs magazine November.
Rural Circuit, Sufi Circuit, Spiritual
Circuit, Ramayana Circuit and Heritage 93. Which of the following correctly
Circuit. (Till 31 Dec, 2018). defines Badland topography?

48
Prelim IAS Test Series (2019) – GS Test 14 (17.03.2019)
GS (Paper-I) Full Test 1
Join Our Telegram Channel https://t.me/UPSCMaterials For Instant Updates

(a) A fairly large area of arable Which of the statements given


land in the irrigated zones of above is/are correct?
India which became saline (a) 1 only
due to over irrigation. (b) 2 only
(b) A region where cultivation is (c) 1 and 2 only
impossible because of the (d) 2 and 3 only
poor fertility of soil.
(c) A region which remained Answer: (b)
uncultivated for more than 5 Explanation:
years. Statement (1) is incorrect.
(d) A region with a large number The World Customs Organization
of deep gullies or ravines. (WCO), established in 1952 as the
Customs Co-operation Council (CCC) is
Answer: (d) an independent intergovernmental body
Explanation: whose mission is to enhance the
Water erosion which is very serious and effectiveness and efficiency of Customs
occurs extensively in different parts of administrations. It is not one of United
India, takes place mainly in the form of Nations' specialized agency.
sheet and gully erosion. Sheet erosion Statement (2) is correct.
takes place on level lands after a heavy World Customs Organization (WCO)is
shower and the soil removal is not easily headquartered in Brussels, Belgium.
noticeable. But it is harmful since it Statement (3) is incorrect.
removes the finer and more fertile top India is a member of WCO and recently a
soil. Gullies deepen with rainfall, cut the four-day regional meeting of the World
agricultural lands into small fragments Customs Organisation (WCO) was held
and make them unfit for cultivation. A at Jaipur with representatives of 33-
region with a large number of deep member countries of Asia attending it.
gullies or ravines is called a badland Source: Vajiram and Ravi current
topography. Hence option (d) is the affairs magazine November.
correct answer.
Source: INDIA: PHYSICAL 95. With reference to the Indian Prime
ENVIRONMENT Page 73 Minister, consider the following
statements:
94. Consider the following statements (1) The Constitution does not
with regard to World Customs contain any specific
Organization (WCO): procedure for the selection
(1) It is a specialized agency of and appointment of the
the United Nations (UN) Prime Minister.
whose mission is to enhance (2) The term of the Prime
the effectiveness and Minister is not fixed and he
efficiency of Customs holds office during the
administrations. pleasure of the President.
(2) It is headquartered in (3) He is the Chairman of the
Brussels, Belgium. National Water Resources
(3) India is not a member of Council.
World Customs Organization Which of the statements given
(WCO). above are correct?

49
Prelim IAS Test Series (2019) – GS Test 14 (17.03.2019)
GS (Paper-I) Full Test 1
Join Our Telegram Channel https://t.me/UPSCMaterials For Instant Updates

(a) 1 and 2 only The consolidation of a kingdom as vast


(b) 2 and 3 only as the Delhi Sultanate needed reliable
(c) 1, 2 and 3 governors and administrators. Rather
(d) None of the above than appointing aristocrats and landed
chieftains as governors, the early Delhi
Answer: (c) Sultans, especially Iltutmish, favoured
Explanation: their special slaves purchased for military
Statement (1) is correct. service, called bandagan in Persian.
The Constitution does not contain any They were carefully trained to man some
specific procedure for the selection and of the most important political offices in
appointment of the Prime Minister. Article the kingdom. Since they were totally
75 says only that the Prime Minister shall dependent upon their master, the Sultan
be appointed by the president. could trust and rely upon them. The
Statement (2) is correct. Khaljis and Tughluqs continued to use
The term of the Prime Minister is not bandagan and also raised people of
fixed and he holds office during the humble birth, who were often their
pleasure of the president. However, this clients, to high political positions. They
does not mean that the president can were appointed as generals and
dismiss the Prime Minister at any time. governors. However, this also introduced
So long as the Prime Minister enjoys the an element of political instability. Hence
majority support in the Lok Sabha, he (a) is the correct answer.
cannot be dismissed by the President. Source: NCERT: Class VII: Our Pasts-
Statement (3) is correct. II: Page No: 37-38.
Prime Minister is the chairman of the
NITI Aayog, National Development 97. Which of the following are the
Council, National Integration Council, remedial measures to reduce soil
Inter-State Council and National Water erosion?
Resources Council. (1) Mixed farming
Source: Indian Polity- M Laxmikanth, (2) Controlled grazing
4th Edition ch-19; PM; page- 19.1 (3) Regulated forestry
(4) Cover cropping
96. ‘Bandagan’ during the medieval Select the correct answer using the
period of Indian history were code given below:
generally: (a) 1 and 2 only
(a) Special slaves purchased for (b) 1, 3 and 4 only
military service (c) 3 and 4 only
(b) Type of a nomadic group who (d) 1, 2, 3 and 4
predominantly engaged in
hunting. Answer: (d)
(c) Peasant serving the kings Explanation:
during Delhi Sultanate period Over-grazing and shifting cultivation in
(d) Minister in charge of religious many parts of India have affected the
and charitable patronage natural cover of land and given rise to
during Akbar’s period. extensive erosion. Contour terracing,
regulated forestry, controlled grazing,
Answer: (a) cover cropping, mixed farming and crop
Explanation: rotation are some of the remedial

50
Prelim IAS Test Series (2019) – GS Test 14 (17.03.2019)
GS (Paper-I) Full Test 1
Join Our Telegram Channel https://t.me/UPSCMaterials For Instant Updates

measures which are often adopted to suffer-wasting-global-hunger-


reduce soil erosion. Hence (d) is the index/article25195277.ece
correct answer.
Source: India: Physical Environment, 99. Consider the following statements
Page 74 with reference to the Rajya Sabha:
(1) The Constitution has fixed
98. Consider the following statements the term of office of a
with regard to Global Hunger Index member of the Rajya Sabha
2018: as six years.
(1) It is published by the (2) The Constitution lays down
International Food Policy that one must be a member
Research Institute (IFPRI). of a Scheduled Caste or
(2) India’s ranking has improved Scheduled Tribe, if he/she
in 2018 compared to 2017. wants to contest a seat
Which of the statements given reserved for them in the
above is/are correct? Rajya Sabha.
(a) 1 only Which of the statements given
(b) 2 only above is/are correct?
(c) Both 1 and 2 (a) 1 only
(d) Neither 1 nor 2 (b) 2 only
(c) Both 1 and 2
Answer: (d) (d) Neither 1 nor 2
Explanation:
Statement (1) is incorrect. Answer: (d)
The Global Hunger Index (GHI) is a tool Explanation:
designed to comprehensively measure Statement (1) is incorrect.
and track hunger globally, regionally, and The Constitution has not fixed the term of
by country. It is released annually by office of members of the Rajya Sabha
Welthungerhilfe and Concern Worldwide. and left it to the Parliament. Accordingly,
The International Food Policy Research the Parliament in the Representation of
Institute was also involved with the the People Act (1951) provided that the
publication until this year. Four main term of office of a member of the Rajya
indicators are used to calculate hunger Sabha shall be six years.
levels undernourishment, child wasting Statement (2) is incorrect.
(low weight for height), child stunting (low It is not the Constitution but the
height for age) and child mortality. Parliament which has laid down the
Statement (2) is incorrect. additional qualification in the
Overall, India has been ranked at 103 out Representation of People Act (1951)
of 119 countries in the Index, with hunger that one must be a member of a
levels in the country categorised as Scheduled Caste or Scheduled Tribe, if
“serious”. Its ranking has dropped three he wants to contest a seat reserved for
places from last year. them in the Rajya sabha.
Source: Vajiram and Ravi current Source: Indian Polity- M Laxmikanth,
affairs magazine October 4th Edition ch-22; Parliament; page-
https://www.thehindu.com/news/natio 22.4
nal/21-of-indian-children-under-5-

51
Prelim IAS Test Series (2019) – GS Test 14 (17.03.2019)
GS (Paper-I) Full Test 1
Join Our Telegram Channel https://t.me/UPSCMaterials For Instant Updates

100. Consider the following statements country, the country must return the
with respect to the Most Favoured favour. Though MFN sounds like special
Nation (MFN) principle: treatment, it actually means non-
(1) It is governed by the World discrimination - treating virtually
Trade Organisation's (WTO) everyone equally. Each member treats
General Agreement on all the other members equally as 'most
Tariffs and Trade (GATT). favoured' trading partners. If a country
(2) It entails special treatment for improves the benefits that it gives to one
some nations, where the trading partner, it has to give the same
granting nation has a trade 'best' treatment to all the other WTO
advantage. members so that they all remain 'most
(3) The MFN status was favoured.
accorded to Pakistan in 1996 Statement (3) is correct.
as per India’s commitments The MFN status was accorded to
as a WTO member. Pakistan in 1996 as per India’s
Which of the statements given commitments as a WTO member. But
above are correct? Pakistan has not reciprocated, reportedly
(a) 1 and 2 only citing “non-tariff barriers” erected by India
(b) 2 and 3 only as well as huge trade imbalance.
(c) 1 and 3 only Recently India has decided to withdraw
(d) 1, 2 and 3 the 'Most Favoured Nation' status given
to Pakistan, in the wake of the worst-ever
Answer: (c) terror attack in Jammu and Kashmir.
Explanation:
Statement (1) is correct.
The Most Favoured Nation (MFN) status
is governed by the World Trade
Organisation's (WTO) General
Agreement on Tariffs and Trade (GATT).
Countries signatory to the agreement
commit against discriminating each other
and rest of the WTO member countries.
Statement (2) is incorrect.
Despite what its name suggests,
bestowing MFN status on a trade partner
doesn’t imply that India gives them
special treatment. The term, in World
Trade Organisation (WTO) parlance,
refers to the principle of non-
discrimination and is a clause that is
applicable to all members of the WTO.
Under the MFN status, a WTO member
country is obliged to treat other trading
nations in a non-discriminatory manner,
especially with regard to customs duty
and other levies. The WTO says that if a
country extends favour to another

52
Prelim IAS Test Series (2019) – GS Test 14 (17.03.2019)
GS (Paper-I) Full Test 1

S-ar putea să vă placă și